• Shuffle
    Toggle On
    Toggle Off
  • Alphabetize
    Toggle On
    Toggle Off
  • Front First
    Toggle On
    Toggle Off
  • Both Sides
    Toggle On
    Toggle Off
  • Read
    Toggle On
    Toggle Off
Reading...
Front

Card Range To Study

through

image

Play button

image

Play button

image

Progress

1/141

Click to flip

Use LEFT and RIGHT arrow keys to navigate between flashcards;

Use UP and DOWN arrow keys to flip the card;

H to show hint;

A reads text to speech;

141 Cards in this Set

  • Front
  • Back
JD is a 63 yo white male who presents to Primary Care for his yearly check-up. He has complaints of urinary frequency, incomplete emptying and mild back pain. Based upon patient’s symptoms, his doctor performs a DRE and finds an irregularity. Blood test reveals PSA of 37. Patient is referred to oncology where further workup reveals biopsy proven prostate cancer with scans showing metastatic disease. Patient is staged as T4N1M1c. He is agreeable to starting any kind of therapy. What is the best option for this patient’s?
a. Active Surveillance
b. Systemic Chemotherapy with Docetaxel and Prednisone
c. LHRH agonist + anti-androgen for first two weeks
d. Radiation therapy alone
c. LHRH agonist + anti-androgen for first two weeks

a. Patient has active disease that warrants treatment
b. Should be reserved for patients with castration recurrent metastatic disease
c. Per NCCN guidelines, treatment should begin immediately in the presence of tumor-related symptoms or overt metastases (category 1 evidence).
d. Patient has T4N1M1c disease, classified as high recurrence risk. RT alone would not be appropriate for high risk disease per NCCN guidelines.
AF is an 87 year old AAM with a past history of DM, osteoporosis, HTN, PVD and COPD. He presents to his primary care physician with complaints of severe leg, back, shoulder and hip pain. Blood test reveals PSA 23. X RAYs are performed and do not reveal any fractures. The patient is referred to oncology. CT and bone scans are performed revealing a mass suspicious for prostate cancer with multiple bone lesions. No other metastatic disease is found. The patient refuses any further work-up and does not want to know if he has cancer, but he would like to address his bone pain. What are AF’s best options for treatment?
a. Watch and wait
b. IV bisphosphonate treatment with Zolendronic acid
c. Systemic chemotherapy with docetaxel and prednisone
d. Orchiectomy
b. IV bisphosphonate treatment with Zolendronic acid

a. Patient with multiple bone lesions that could cause pathologic fractures, watching and waiting will not provide pain relief
b. NCCN guidelines recommend that patients with bone metastases receive zolendronic acid q3-4 weeks to prevent disease-related skeletal complications including pathological fractures and spinal cord compression
c. Patient refuses further work-up, has several co-morbidities and advanced age. The risk of complications and side effects from chemotherapy may outweigh any benefit that he would receive from treatment.
d. Patient refuses further work-up, has several co-morbidities and advanced age. The risk of complications from surgery would outweigh the benefits that he would receive from therapy.
A drastic increase in prostate cancer diagnosis in the early 1990s is believed to be the result of what?
a. Increased awareness through television and print ads
b. More men receiving DREs with their yearly physicals
c. TAX trial
d. PSA utilization
d. PSA utilization
Chemoprevention with 5-alpha reductase inhibitors such as finasteride eliminates the risk of developing prostate cancer and should be used for at least 9 years.
True
False
False

The do not eliminate the risk of developing PC, only reduce the clinical incidence.
There is no evidence beyond 7 years of use.
A.S. is a healthy, 49 year old female who presented to her primary care physician with the chief complaint of intermittent abdominal pain, constipation and bloating.  Initial work-up with colonoscopy revealed a nonobstructing tumor 32 cm from the anal verge. A diagnostic CT scan of the chest, abdomen, and pelvis did not demonstrate evidence of metastatic disease. CEA at time of surgery was 4253 ng/ml.  The patient underwent a transverse colectomy which revealed a mass invading through the muscularis propria into the subserosa, with 6 positive lymph nodes and no evidence of metastases.  Final pathology revealed a moderately differentiated adenocarcinoma and T3 N2 M0 Stage IIIC colon cancer.
Labs:
WBC   8.3 k/mcL Scr       1.2 mg/dl Hgb      12.7 g/dl BUN    15 mg/dl
Hct       35% Bili       0.7 mg/dl Plt        180 k/mcL

Which one of the following statements regarding treatment of this patient is correct?
a. No therapy is required because a survival benefit has not been seen with adjuvant therapy.
b. Adjuvant therapy with 5-fluoruracil/leucovorin chemotherapy is the treatment of choice.
c. Adjuvant therapy should consist of oxaliplatin-based chemotherapy, which is superior to 5-fluoruracil based regimens.
d. Irinotecan in combination with bolus 5-fluoruracil/leucovorin (given as the IFL regimen) is treatment of choice.
c. Adjuvant therapy should consist of oxaliplatin-based chemotherapy, which is superior to 5-fluoruracil based regimens.

a. Incorrect.  Results from a meta-analysis of 5 trials in which patients with Stage II and Stage III colon cancer were randomly assigned to receive surgery alone or surgery followed by adjuvant chemotherapy demonstrated that most of the benefit of adjuvant therapy was seen in patients with Stage III disease.  These trials include the MOSAIC, X-ACT, NSABP C-07, CALGB C89803 and PETACC-3.
b. Incorrect. The MOSAIC trial evaluated the efficacy of FOLFOX4 compared to 5-FU/LV in the adjuvant setting in 2246 patients. Completely resected Stage III patients had improved disease free survival (DFS) at 5 years and overall survival (OS) with FOLFOX4. Based on these results, FOLFOX 4 is recommended as treatment for early stage colon cancer. 5-FU/LV is an alternative in patients with comorbid illness, poor performance status (PFS) or tolerability/ logistical issues. It is not considered treatment of choice.
c. Correct. FOLFOX (oxaliplatin, 5FU + leucovorin) is superior for Stage III patients. The MOSAIC trial randomized patients to receive either FOLFOX4 or 5-FU/LV. In stage III patients, DFS and OS was significantly better for FOLFOX4 that 5-FU/LV. Toxicities include neutropenia > grade 3, diarrhea and paresthesias in FOLFOX4 and considered tolerable.
d. Incorrect. Weekly bolus IFL should not be used as adjuvant therapy in colon cancer. The US Intergroup trial CALGB C89803 evaluated irinotecan plus bolus 5-FU/LV (IFL regimen) vs 5-FU/LV alone in Stage III colon cancer. The results indicated no improvement in terms of overall survival or failure free survival for IFL. It was also associated with a greater degree of neutropenia, fever and death.
R.L. has Stage IV colon cancer with metastasis to the liver. He returns to the clinic 4 months after finishing first line therapy and a CT scan shows 2 new pulmonary lesions. KRAS mutation analysis using DNA from the primary tumor cells is performed and shows a KRAS mutation.

Which agent can not be used as second line therapy due to the KRAS mutation?
a. Cetuximab
b. Rituximab
c. Irinotecan
d. Bevacizumab
a. Cetuximab

Correct. Cetuximab is a monoclonal antibody that blocks epidermal growth factor receptor (EGFR). About 40% of patients with metastatic colon cancer have tumors with a mutated form of the KRAS gene, and these patients are unlikely to respond to treatment with Cetuximab and Panitumumab. Patients with the normal, or wild-type, KRAS genes are likely to respond to these drugs.
b. Incorrect. Rituximab is a monoclonal antibody directed against the protein CD20 found on B-cells in Non- Hodgkin’s Lymphoma. It has no activity in colon cancer.
c. Incorrect. Irinotecan is a camptothecin analogue that inhibits topoisomerase I. It is not affected by KRAS mutations. It is given in combination (FOLFIRI) with cetuximab as second line treatment, but it is not affected by KRAS mutations.
d. Incorrect. Bevacizumab is a monoclonal antibody that binds to and inhibits Vascular Epithelial Growth Factor (VEGF). It is not affected by KRAS mutations.
H.K. is a 67 year old woman with multiple myeloma.  She received her seventh dose of Velcade (bortezomib) 2 days ago.  Her family has brought her to the ER today because she has new fevers and has become confused and drowsy during the last 12 hours.   Her labwork and vital signs are as follows:

Temp = 39.2 C HR = 116 RR = 25 BP = 84/36

WBC = 0.58 ANC = 0.15 Na = 140 K = 3.7
Cl = 101 CO2 = 28 BG = 114 BUN = 65
SrCr = 2.9 Ca = 8.4 Phos = 2.9 Mag = 1.9
TBili = 0.3 AST = 45 ALT = 52 LDH = 810

A chest Xray and blood cultures have been performed, and are pending.

H.K. is unable to relay to you if she has any other complaints, as she is currently difficult to arouse.  Her family reports that she does not have any drug allergies, and that she is currently taking acyclovir, gabapentin and dexamethasone.

Based on this information, what drug therapy do you recommend for HK?
I. Cefepime
II. Piperacillin + tazobactam
III. Tobramycin
IV. Vancomycin
V. Filgrastim

a. I and II
b. I, II and IV
c. II and IV
d. I, III and IV
e. I, III, IV and V
e. I, III, IV and V

Answer:  E.  The students should recognize that this is a neutropenic patient presenting with sepsis (tachcardic, hypotensive, mental status changes and declining renal function).  She would need double coverage for gram negative organisms (cefepime + tobramycin OR pip/tazo + tobramycin) as well as gram positive coverage (vancomycin).  Because she is septic, she should also receive a growth factor (filgrastim).  I would give full credit for an answer of E, but partial credit for D as the growth factor would be more of a judgement call.
Your neutropenic patient has had persistent fevers despite your initial antimicrobial choices.  He is asymptomatic and all of the cultures remain negative to date.  At what point should an antifungal agent be added to the empiric regimen?
a. When the patient’s fevers exceed 40.0 C
b. After 2 days of broad spectrum antibiotics
c. After 3 days of broad spectrum antibiotics
d. After 4-5 days of broad spectrum antibiotics
e. Antifungal agents are not necessary since all cultures are negative and he remains asymptomatic
d. After 4-5 days of broad spectrum antibiotics

Answer: D.  Current recommendations are to add an antifungal agent if the patient remains febrile for 4-5 days after initial therapy.
You have determined that a chemotherapy regimen is highly emetogenic.  Which combination of prophylactic agents would be the MOST appropriate for the patient’s first cycle of these agents?
I. Ondansetron
II. Lorazepam
III. Dexamethasone
IV. Aprepitant
V. Dronabinol

a. I only
b. I and II
c. I, II and III
d. I, III and IV
e. I, IV and V
d. I, III and IV

Answer:  D.  Current guidelines recommend a serotonin antagonist + dexamethasone +/- aprepitant for prophylaxis of acute CINV from highly emetogenic chemotherapy.
Which of the following statements regarding the treatment of breakthrough CINV is true?
a. Prevention of breakthrough CINV is imperative to preventing it
b. Use an additional agent from a different drug class
c. Consider scheduling the additional agent, instead of PRN
d. Consider a route other than oral
e. All of the above
e. All of the above

Answer:  E.  All of the choices are principles that should be considered for breakthrough CINV.
E.K. is a 64 yo male who presents to the clinic for routine follow-up for CML, diagnosed 10 years ago. He complains of increased fatigue, dyspnea on exertion and a “cold” that doesn’t want to go away. His PMH is significant for hypertension and CML. His current medications include lisinopril 20 mg PO daily, atenolol 25 mg PO daily, simvastatin 20 mg PO QHS and imatinib 400 mg PO daily. His laboratory values are significant for:

8.3 140 109 25 102 uric acid 9.3 mg/dL
113 4.7 30 1.2 LDH 432 IU/L

A repeat bone marrow biopsy shows 21% blasts.

What is the most appropriate therapy for this patient?
a. Increase imatinib to 800 mg PO daily as he is not responding to 400 mg PO daily.
b. Change his therapy to dasatinib 100 mg PO daily as he is failing imatinib.
c. Admit him to the hospital for induction chemotherapy.
d. Decrease the dose of imatinib to 200 mg PO daily as the patient is experiencing signs of toxicity associated with the interaction between simvastatin and imatinib.
c. Admit him to the hospital for induction chemotherapy.

a. Incorrect. The patient has transformed to a blast crisis and now has an acute leukemia. CML in blast crisis is typically resistant to imatinib and requires treatment with conventional chemotherapy.
b. Incorrect. While the patient is considered to be failing imatinib, he has transformed to a blast crisis and now has an acute leukemia which requires treatment with conventional chemotherapy. The patient may be started on dasatinib in addition to chemotherapy, however, single agent dasatinib will not be beneficial in this patient.
c. Correct. The patient has transformed to a blast crisis and now requires induction chemotherapy for treatment of an acute leukemia. The patient will be admitted to the hospital for proper monitoring. Dasatinib or nilotinib may be initiated in combination with chemotherapy.
d. Incorrect. Imatinib is an inhibitor of CYP3A4 and the likely scenario would be increased effects of simvastatin. Toxicity associated with imatinib would include myelosuppression; you would not expect an increased WBC.
F.S. is a 34 yo male recently diagnosed with AML with t(8;21). He presented with symptomatic pancytopenia (WBC 1.5, Hgb 6.3 and platelets 99) and has no other PMH. He is currently day 17 s/p chemotherapy, which consisted of cytarabine 100 mg/m2 days 1-7 and idarubicin 12 mg/m2 days 1-3, and is receiving cefepime 2 g IV q8h, acyclovir 400 mg PO BID, pantoprazole 40 mg PO daily, and a nicotine patch, 21 mg daily. His day 14 bone marrow biopsy showed evidence of residual leukemia.

Which statement most accurately predicts the prognosis for FS in regard to AML?
a. Favorable, relating to his young age, t(8;21), WBC 1.5 at diagnosis and de novo AML.
b. Unfavorable, relating to his young age, t(8;21), WBC 1.5 at diagnosis and de novo AML.
c. Favorable, relating to his young age and early initiation of broad spectrum antibiotics.
d. Unfavorable, relating to t(8;21) and evidence of residual disease on his day 14 bone marrow biopsy.
a. Favorable, relating to his young age, t(8;21), WBC 1.5 at diagnosis and de novo AML.

a. Correct. Age < 55 years, t(8;21), WBC < 25,000 at diagnosis and de novo AML have all been found to be favorable prognostic indicators for patients with AML.
b. Incorrect. Age < 55 years, t(8;21), WBC < 25,000 at diagnosis and de novo AML have all been found to be favorable prognostic indicators for patients with AML.
c. Incorrect. Age < 55 years is a favorable prognostic factor for AML, however, timing of initiation of antibiotics is not correlated to outcomes for patients with AML.
d. Incorrect. Translocation (8;21) is a favorable prognostic factor and results from the day 14 bone marrow biopsy do not correlate with treatment outcomes.
Which of the following statements is FALSE?

a. Gemtuzumab is a recombinant monoclonal antibody which targets CD33, an antigen commonly expressed on myeloid blasts.
b. Common adverse effects of ATRA include increased LFTs, pancreatitis, hypersensitivity and coagulopathies.
c. Adverse effects related to cytarabine are dose dependent. Common adverse effects associated with high dose cytarabine include chemical conjunctivitis and neurotoxicity.
d. Patients receiving alemtuzumab, a monoclonal antibody directed against the CD52 antigen, should always receive prophylaxis for HSV and PCP.
b. Common adverse effects of ATRA include increased LFTs, pancreatitis, hypersensitivity and coagulopathies.

a. Incorrect. This statement is true. Gemtuzumab is a recombinant monoclonal antibody which targets the CD33 antigen. CD33 is expressed on blast cells in >80% of patients with AML.
b. Correct. This statement is false. Increased LFTs, pancreatitis, hypersensitivity and coagulopathies are common adverse effects of l-asparaginase. Common adverse effects of ATRA include rash, dry skin, headache, increased triglycerides, differentiation syndrome, bone pain and flushing.
c. Incorrect. This statement is true. High dose cytarabine is associated with a chemical conjunctivitis and neurotoxicity, which is not seen with lower doses of cytarabine. Patients receiving high dose cytarabine receive prophylactic steroid eye drops and routine neuro-checks.
d. Incorrect. This statement is true. Alemtuzumab is a monoclonal antibody directed against the CD52 antigen which is expressed on T-cells. Depletion of T-cells places the patients at a very high risk for developing opportunistic infections including HSV and PCP. All patients receiving alemtuzumab receive prophylaxis against HSV and PCP.
Which of the following disease states is correctly matched with the common complications?
a. Chronic Myeloid Leukemia – leukostasis, tumor lysis syndrome, differentiation syndrome
b. Chronic Lymphocytic Leukemia – pure red cell aplasia, auto-immune hemolytic anemia, hypogammaglobulinemia
c. Acute Myeloid Leukemia – leukostasis, tumor lysis syndrome, differentiation syndrome
d. Acute Lymphocytic Leukemia – disseminated intravascular coagulopathy, tumor lysis syndrome, hypogammaglobulinemia
b. Chronic Lymphocytic Leukemia – pure red cell aplasia, auto-immune hemolytic anemia, hypogammaglobulinemia

a. Incorrect. CML is commonly associated with leukostasis. Tumor lysis syndrome is a common complication of the acute leukemias and differentiation syndrome is a complication associated with the treatment of APL.
b. Correct. Patients with CLL commonly suffer from pure red cell aplasia, auto-immune hemolytic anemia or hypogammaglobulinemia.
c. Incorrect. Tumor lysis syndrome is a very common complication with all types of AML. Differentiation syndrome is associated with APL, a subtype of AML. However, leukostasis is not a common complication of acute leukemias. This complication is more commonly associated with CML.
d. Incorrect. DIC is a common complication associated with AML and hypogammaglobulinemia is a common complication associated with CLL. Tumor lysis syndrome is a typical complication seen when patients receive induction chemotherapy for ALL.
B.L. is a 27 year old male who presents to primary care complaining of a hard lump on his right armpit. He complains of drenching night sweats, fevers and a 15 pound weight loss over the past 6 weeks. A CT scan is ordered and reveals mediastinal lymphadenopathy as well as a mass in the right axilla. BL is referred to oncology who determines that he has Diffuse Large B Cell Lymphoma. The most likely chemotherapy regimen that BL will receive is:
a. BEACOPP
b. MOPP
c. ABVD
d. R-CHOP
d. R-CHOP

Gold Standard Regimen for DLBCL (non-hodgkins lymphoma)
___________________________ is considered an oncologic emergency causing renal failure, electrolyte abnormalities and possibly death. This is often a consequence of lymphoma treatment in a patient with bulky disease.
a. Febrile Neutropenia
b. B Symptoms
c. Tumor Lysis Syndrome
d. Chemotherapy induced nausea and vomiting
c. Tumor Lysis Syndrome
K.T. is a 57 year old WF who has been experiencing hot flashes for 13 months. She also complains of vaginal dryness that is significantly interfering with her ability to be intimate with her husband. Her PMH is significant for hypertension, osteoarthritis, and GERD. Her current medications include losartan 50mg QD, ibuprofen 400mg TID, and Omeprazole 20mg QD. KT’s last menstrual period was 7 months ago. KT’s physician decides to initiate Climara (17β-estradiol) 0.05mg applied once weekly.

All of the following are true regarding estrogen therapy EXCEPT?
a. Transdermal estrogen is less effective than oral estrogen for treating hot flashes
b. Unopposed estrogen should be avoided in a woman with an intact uterus
c. Adverse effects (such as breast tenderness, nausea) may be reduced with transdermal estrogen
d. A trial of at least 4-8 weeks should be considered before considering hormone therapy as a treatment failure.
e. All of the above are true.
a. Transdermal estrogen is less effective than oral estrogen for treating hot flashes

a. Correct answer. This is False; transdermal and oral estrogen have similar efficacy in treating symptoms
b. This is true; a progestin must be given for at least 10 days of the cycle to avoid endometrial hyperplasia/cancer
c. This is true. Mentioned in lecture along with slide 19.
d. This is true. Mentioned in lecture along with slide 16.
Which of the following is NOT a contraindication to hormone therapy?
a. History of DVT
b. History of MI
c. History of breast cancer
d. History of colorectal cancer
e. History of liver disease
d. History of colorectal cancer

Correct answer. CI to hormone therapy can be found on slide 20 from lecture.
KT has begun hearing rumors that estrogen is “bad for her” and is considering discontinuing therapy. Which of the following is NOT true regarding the estrogen only arm of the WHI?
a. Risk of stroke was increased in the estrogen group
b. Risk of DVT was increased in the estrogen group
c. Risk of colorectal cancer was decreased in the estrogen group
d. Rate of hip fracture was reduced in the estrogen group
e. Risk of invasive breast cancer was lower in the estrogen group
c. Risk of colorectal cancer was decreased in the estrogen group

c. Correct answer: there was no difference between groups on colorectal cancer risk which was different from the WHI est/progestin arm.
d. One of the few approved indications for estrogen therapy is osteoporosis prevention in postmenopausal women.
e. Was lower, although not statistically significant.
P.U. is a 33 year old WF who presents to the Family Medicine Center desiring to start oral contraceptives. A ROS indicates a family history of breast cancer (aunt), diabetes (grandfather), and colorectal cancer (father). Her PE is essentially unremarkable demonstrating a WN WF with NKDA. Her PMH includes a previous contraception attempt with medroxyprogesterone that was dc’d due to irregular bleeding. She is not currently sexually active, but plans to be soon.
Baseline lab data revealed the following:
TC= 180 mg% AST = 14 u/L
LDL=130 mg% ALT = 18 u/L
HDL = 40 mg% BG (non-fasting) = 89 mg%
TG = 155 mg% Serum Fe = 110 mcg/dl

Based on the patient’s previous birth control history and your desire to start her on a “balanced” progestin and estrogen combination pill, which of the following choices makes the most sense for PU?
a. Micronor
b. Demulen 1/35
c. Ortho-Novum 1/50
d. Ortho-Cept
c. Ortho-Novum 1/50
Thirteen weeks after starting the patient on a norethindrone-based combination pill, she returns to the office with a CC of BTBS on day 11 of her cycle.
Which of the following statements is true?
a. Switching her to a tri-phasic pill will solve her problem
b. Tell her that her problem is due to a progestin deficiency
c. Tell her to keep taking her current pill since it is likely that her problem will resolve spontaneously
d. Switch her to a more estrogenic progestin like ethynodiol diacetate.
d. Switch her to a more estrogenic progestin like ethynodiol diacetate.
After finally stabilized and content on her BCP, PU has time to read the package insert that her pharmacist keeps handing her when she picks up her refills. After reading the inserts she has an anxiety attack and calls you for clarification. Which of the following statements that she says she read is actually true?
a. “My blood pressure will go up 10 mm Hg systolic and 5 mm Hg diastolic”
b. “My risk for gallbladder disease is double if I keep using my pill”
c. “My risk of cervical and endometrial cancer is higher if I stay on my pill”
d. “The insert said that my iron levels will go drop if I stay on the pill”
b. “My risk for gallbladder disease is double if I keep using my pill”
LB is a 69 year old postmenopausal female with osteoporosis. She recently experienced a vertebral fracture and suffers from considerable pain related to this fracture. The patient was recently started on alendronate. Which of the following is NOT an appropriate monitoring parameter in LB?
a. Perform a repeat DXA scan in 6 months
b. Measure the patient’s height at subsequent clinic visits
c. Check the patient’s renal function periodically
d. Evaluate whether the patient has experienced Gi side effects, such as esophagitis
a. Perform a repeat DXA scan in 6 months (Objective 4, Case Study Question #5) All of the following are recommended monitoring strategies except for repeating the DXA at 6 months. A minimum of 24 months is required to accurately detect a difference in BMD in patients starting BP treatment. Checking the BMD at more frequent intervals is not recommended since changes may be related to standard deviation in BMD calculation and may be misleading.
LB was prescribed ibandronate (Boniva) 150 mg once. Which of the following is most correct with respect to counseling LB?
a. Take ibandronate 150 mg at least 30 minutes before any other food, beverage, or medication
b. Calcium supplements are no longer needed and may be discontinued once ibandronate is started
c. Stop taking the drug and contact your doctor if you develop difficulty or pain on swallowing
d. If you forget to take a dose until after you have eaten, take it as soon as you remember later that same day
c. Stop taking the drug and contact your doctor if you develop difficulty or pain on swallowing

(Objective 5, Case Study Question #6) Ibandronate must be taken 60 minutes before other medications, food and beverages – not 30 minutes like other bisphosphonates. Adequate intake of calcium and vitamin D are pivotal to ensuring BP efficacy; however, their administration must be separated from BP administration. If a dose of ibandronate 150 mg is forgotten it should not be taken later that same day due to the potential for drug-food interaction. Instead, the day’s dose should be skipped entirely and the normal dosing schedule should be resumed the subsequent morning.
A.B. is a 73-year-old white female who presents to her primary care doctor for an annual physical. Her medical history is rather unremarkable. She is concerned that she has osteoporosis because both her mother and younger sister experienced premature fragility fractures. She correctly calculated her 10-year risk of fracture with FRAX to be 4% at the hip. Which of the following interventions would be most appropriate at this time?
a. Encourage lifestyle modification and measure AB’s bone mineral density in 2 years
b. Encourage lifestyle modification and start risedronate (Actonel™) 35 mg once a week
c. Encourage lifestyle modification and start calcitonin (Miacalcin™) 200 IU in alternating nostrils once daily
d. Encourage lifestyle modification and start teriparatide (Forteo™) 20 mcg subcutaneously once daily
b. Encourage lifestyle modification and start risedronate (Actonel™) 35 mg once a week

(Objective 3, FRAX calculation and Who to Treat slides from lecture) B is the only correct answer. Pharmacological therapy, in addition to lifestyle modification, is clearly indicated in this patient given the presence of multiple risk factors and vertical height loss, which in all likelihood signifies the presence of silent vertebral fractures. According to AACE, a T-score < -1.5 plus risk factors indicates pharmacological therapy. In the absence of contraindications (none are present in this patient), a bisphosphonate is considered first line. Calcitonin is not a first line agent due to limited efficacy for preventing nonvertebral fractures. Teripratide is cost prohibitive and would only be in patients who fail a bisphosphonate or have severe osteoporosis and multiple fractures at baseline.
J.H. is a 78 yo patient admitted to the hospital from a nursing home for evaluation after he developed a pneumonia and began to cough up blood. He is a partially paralyzed and bed-bound after having a series of strokes two years ago. After a work-up of his pneumonia, he is diagnosed with metastatic non-small cell lung cancer (adenocarcinoma, T3, N0, M1) that is causing a post-obstructive pneumonia.

Which of the following is the best choice for treatment in this patient?
a. Neoadjuvant chemotherapy followed by surgical resection of his tumor
b. Initial surgical resection, followed by 6 cycles of adjuvant chemotherapy with paclitaxel/carboplatin
c. Combination therapy with paclitaxel, carboplatin, and bevacizumab
d. Single agent docetaxel
d. Single agent docetaxel

Answer: D. A and B are incorrect; since JH has metastatic disease, surgical resection is not an option for him. In addition, he is likely to have a poor performance status, which renders him a poor surgical candidate. While the addition of bevacizumab to paclitaxel/carboplatin improved the response rates and survival compared to chemotherapy alone for patients with advanced stage NSCLC, bevacizumab use should be avoided in this patient since he presented with hemoptysis. Therefore, option C incorrect. That leaves option D as the best answer.
In the TRIBUTE study evaluating the addition of the EGFR inhibitor erlotinib to paclitaxel/carboplatin, the addition of erlotinib to chemotherapy did not offer a survival advantage, except for the subset of patients that were thought to have tumors that demonstrated EGFR mutations. Which of the following is NOT thought to be a surrogate for EGFR mutations?
a. History of never smoking
b. HER2 overexpression
c. Female gender
d. Patients of Asian origin
b. HER2 overexpression

Answer: B. In the TRIBUTE study, only the subset of patients who had never smoked experienced a survival advantage from the inclusion of erlotinib. From other trials of both EGFR inhibitors (erlotinib and gefitinib), subset analyses consistently show survival benefits in patients who are: non-smokers (particularly never-smokers), women, and patients of Asian background. Option B is the correct answer, as HER2 overexpression is not predictive in lung cancer patients.
A.P. is a 59 yo WF who was recently found, on routine screening mammography, to have a 2 cm spiculated mass in her left breast. On review of her reproductive history, she experienced menarche at age 12, has had two pregnancies, both resulting in live births (1st delivery at age 22), no history of hormone exposure, no family history of breast cancer. She is post-menopausal. She was referred for further evaluation of her breast mass. An ultrasound revealed a hypoechoic, irregular spiculated mass. A fine needle biopsy was positive for malignancy.

She was referred to an oncologic surgeon, and underwent wide local excision; the pathology was positive for an infiltrating lobular carcinoma. She also underwent an axillary lymph node dissection, which revealed that 3 of 11 lymph nodes were positive, with limited extranodal extension. The ER value was 135 fmol/mg, and PR value was 96 fmol/mg. Her staging workup indicated that she has a Stage IIB breast cancer.

Which of the following approaches is most appropriate for AP?
a. Doxorubicin + cyclophosphamide followed by paclitaxel (AC -> T), along with tamoxifen
b. Doxorubicin + cyclophosphamide followed by paclitaxel (AC -> T), along with anastrazole
c. Cyclophosphamide, methotrexate, and fluorouracil (CMF), along with tamozifen
d. Cyclophosphamide,methotrexate, and fluorouracil (CMF), along with anastrazole
b. Doxorubicin + cyclophosphamide followed by paclitaxel (AC -> T), along with anastrazole

Answer: B. While the CMF regimen can be used in the adjuvant setting, it is generally reserved for patients with node negative disease. Since AP has positive lymph nodes, she is better served by getting the AC -> taxane regimen for her initial adjuvant therapy; this makes options C and D incorrect. Since her tumor is positive for hormone receptors, she should also receive adjuvant hormonal therapy. Although tamoxifen may be used, the ASCO technology assessment from 2004 indicates that post-menopausal therapy should receive an aromatase inhibitor, such as anastrazole. Therefore, of the listed approaches, B is the most appropriate.
T.J. is a 35 yo premenopausal woman presenting with newly diagnosed left breast cancer. Two months ago, she noticed that her left nipple was retracted and discovered a lump during a self-exam. A diagnostic mammogram revealed a 2.5 cm speculated mass in her left breast without calcifications. She underwent a lumpectomy and lymph node dissection, which revealed involvement in 2 of 14 nodes. Final pathology confirmed a 2.5 cm primary mass that was positive for ER and PR expression and negative for HER2 overexpression by FISH. She is diagnosed with a Stage IIB breast cancer. Her physician has recommended that she receive dose-dense chemotherapy with doxorubicin/cyclophosphamide given every 2 weeks for 4 cycles, followed by paclitaxel every 2 weeks for 4 cycles (dose dense AC -> T). Since her tumor was HR-positive, she will need adjuvant hormonal therapy in addition to her chemotherapy.

Which of the following statements is TRUE?
a. She should receive tamoxifen 20 mg PO daily
b. She should receive letrozole 2.5 mg PO daily
c. She should be given paroxetine to manage her hot flashes
d. She should receive adjuvant hormonal therapy indefinitely to prevent disease recurrence
b. She should receive letrozole 2.5 mg PO daily

Answer: A. Because she is premenopausal, she should receive an anti-estrogen such as tamoxifen. While aromatase inhibitors, such as letrozole, as very effective in post-menopausal patients, they will not block ovarian estrogen production in a patient who is still having menstrual cycles, and should be avoided. Therefore B is incorrect. While the SSRIs can be effective in controlling hot flashes, paroxetine can inhibit cytochrome P450 2D6, which is one of the enzymes responsible for the metabolism of tamoxifen; paroxetine should be avoided in patients being treated with tamoxifen, which makes option C incorrect. Option D is incorrect as well; the standard approach is to use adjuvant hormonal therapy for a total of five years. While the NSABP B-14 study showed an increase in disease-free survival with 10 years of tamoxifen, there was no improvement in overall survival; to date there is no data to support indefinite, continued hormonal therapy.
B.L. is a 55 yo male with newly diagnosed head and neck cancer. He is scheduled to get neoadjuvant chemotherapy with cisplatin, methotrexate, and fluorouracil, followed by a 2 week course of radiation therapy.

Which of the following statements is false?
a. Neoadjuvant therapy is given prior to surgery
b. Neoadjuvant therapy is given after surgery
c. Neoadjuvant therapy may allow downstaging of the primary tumor
b. Neoadjuvant therapy is given after surgery

Answer: B. Neoadjuvant therapy is given prior to surgery to reduce the tumor size. This may reduce the extent of surgery needed, and may allow a tumor to be downstaged (i.e. a tumor that is stage III before therapy may decrease to a stage II after therapy, making it more amenable to surgical resection.
LM is a 63 yo female with newly diagnosed acute myeloid leukemia (AML) who was given induction therapy with cytarabine + idarubicin (7+3). She finished her chemotherapy yesterday and is generally feeling better. She has been getting up and walking around the nursing unit with assistance, but has fallen several times when attempting to walk on her own.

Which of the following are concerns regarding LM?
a. She is at increased risk of infection, and should be allowed to ambulate only while wearing a mask.
b. She may be experiencing neurotoxicity, and should be closely monitored for worsening cerebellar symptoms.
c. She may be having bladder toxicity, and should be watched closely for the development of severe hematuria.
d. A and B
e. B and C
f. All of the above
d. A and B

Answer: D. Because her counts will have fallen after receiving her leukemia induction chemotherapy, she does have an increased risk of infection. Her falls are concerning for cerebellar toxicity, and she should be closely monitored for signs of worsening toxicity. C. is false, as the regimen LM received is not associated with bladder toxicity.
S.K. is a 65 yo female diagnosed last year with locally advanced NSCLC. She received initial therapy with paclitaxel/carboplatin, which she did not tolerate well due to severe arthralgias. Her therapy was changed to gemcitabine/cisplatin, which was better-tolerated. Disease staging evaluation done after completion of her last cycle of chemotherapy indicated that her lung cancer had not improved despite chemotherapy. She wishes to pursue further therapy and has been searching the internet, and asks about trying Tarceva (erlotinib).

Which of the following is true regarding erlotinib?
a. It targets the Bcr-Abl tyrosine kinase
b. It targets the epidermal growth factor receptor (EGFR) tyrosine kinase
c. It target the HER2 receptor
d. It is indicated for first line therapy for NSCLC
b. It targets the epidermal growth factor receptor (EGFR) tyrosine kinase

Answer: B. It targets the EGFR tyrosine kinase, not the Bcr-Abl tyrosine kinase or the HER2 receptor. It is indicated in patients who have failed at least one prior chemotherapy regimen. In the general population, the addition of erlotinib to chemotherapy as first line therapy does not produce a survival benefit compared to chemotherapy alone.
V.B. is a 54 yo female recently diagnosed with non-Hodgkin’s Lymphoma. She is scheduled to begin chemotherapy with CHOP (cyclophosphamide, doxorubicin, vincristine, and prednisone) + Rituxumab (anti-CD20 monoclonal antibody) x 6 cycles.

Given her planned chemo regimen (CHOP), what major organ toxicity is she at risk for?
a. Acute renal failure
b. Hearing loss
c. Congestive heart failure
d. Acute cerebellar toxicity
c. Congestive heart failure

Answer: C. Cardiac toxicity is a serious and potentially life-threatening toxicity seen with anthracyclines, including doxorubicin. This regimen is unlikely to cause significant renal failure, central nervous system toxicity, or ototoxicity.
An 18 year old patient with no prior medical history and no known drug allergies was brought into the Trauma Center for multiple gunshot wounds to the abdomen. Patient went for emergency exploratory laparotomy. Patient injuries consisted of a gastric injury, hepatic artery and duct injury, large colon injury, jejuna injury, left diaphragm injury, and a left pneumothorax. Patient was repaired operatively and transferred to the trauma unit for aggressive medical and ventilator management. Seven days after the operation patient became febrile (101.3◦), tachycardia (123) with leucocytosis of 23 with 10% Bands. Patient was placed on continuous renal replacement therapy (CRRT) for acute renal failure. Blood and fungal cultures were drawn and empiric antibiotics (Pipercillin/tazobactam 3.375 gm IV q6h, Vancomycin 1.25gm IV q12 hours, and an antifungal agent) were started. Assume normal renal function while on CRRT.

What would you recommend as initial antifungal therapy?
a. Liposomal Amphotericin B IV at 5 mg/kg/day
b. Fluconazole 800 mg IV X 2 days, then 400 mg IV daily
c. Voriconazole 6 mg/kg IV every 12 hours X 2 doses, then 4 mg/kg IV every 12 hours
d. Fluconazole 800 mg IV X 1 day, then 400 mg IV daily plus flucytosine 25 mg/kg po divided every 6 hours
e. Conventional Amphotericin B IV at 0.6 mg/kg/day
b. Fluconazole 800 mg IV X 2 days, then 400 mg IV daily
Three months later this patient is still recovering in the Trauma Unit. Her course was unfortunately complicated by the development of several gastrointestinal fistulas which required daily abdominal washouts and chronic parenteral nutrition via central line. Patient deteriorated over the course of the day becoming febrile and hypotensive. The patient required fluid boluses, norepinephrine, and increased ventilator support. Patient’s renal function returned to base-line; however, her hepatic function tests show severe dysfunction. Blood and fungus cultures were drawn. The following antibiotic regimen was started for bacteremia: Cefepime 2 gm IV every 12 hours, Gentamicin 300 mg IV every 24 hours, and Linezolid 600 mg IV every 12 hours.

At this time, what would you like to start for possible fungemia?
a. Caspofungin 70 mg IV every 24 hours
b. Fluconazole 800 mg IV X 1 days, then 400 mg IV daily
c. Voriconazole 6 mg/kg IV every 12 hours
d. Anidulafungin 200 mg IV X 1 day, then 100 mg IV daily
e. Liposomal Amphotericin B IV at 1 mg/kg/day
d. Anidulafungin 200 mg IV X 1 day, then 100 mg IV daily
Blood and Fungal culture with sensitivities report back as the following:
Blood: Vancomycin-resistant enterococcus (VRE) Fungal: Candida parapsilosis
Ampicillin: Reistant Fluconazole: Sensitive
Gentamicin synergy: Sensitive Flucytosine: Sensitive
Vancomycin: Resistant Amphotericin B: Sensitive
Linezolid: Sensitive

Based on fungal culture and sensitivity results, what would you like to do with your antifungal regimen?
a. Caspofungin 35 mg daily
b. Anidulafungin 100 mg daily
c. Liposomal Amphotericin B IV at 5 mg/kg/day
d. Fluconazole 400 mg IV daily
e. Voriconazole 4 mg/kg IV every 12 hours
d. Fluconazole 400 mg IV daily
AB is a 71-year-old white female who presents to her primary care doctor for an annual physical. Her medical history is rather unremarkable. Her only complaint is that she feels she is “getting shorter” and her posture is getting worse. She denies a history of any fracture. The physical exam is remarkable for 3 inch height loss. Dual-energy x-ray absorptiometry of the hip shows a T score of –1.6.

Which of the following interventions would be most appropriate?
A. Encourage lifestyle modification and reexamine bone mineral density in 2
years
B. Encourage lifestyle modification and start alendronate (FosamaxTM) 70 mg once a week
C. Encourage lifestyle modification and start calcitonin (MiacalcinTM) 200 IU in alternating nostrils once daily
D. Encourage lifestyle modification and start teriparatide (ForteoTM) 20 mcg subcutaneously once daily
B. Encourage lifestyle modification and start alendronate (FosamaxTM) 70 mg once a week

B is the only correct answer. Pharmacological therapy, in addition to lifestyle modification, is clearly indicated in this patient given the presence of multiple risk factors and vertical height loss, which in all likelihood signifies the presence of silent vertebral fractures. According to AACE, a T-score < -1.5 plus risk factors indicates pharmacological therapy. In the absence of contraindications (none are present in this patient), a bisphosphonate is considered first line. Calcitonin is not a first line agent due to limited efficacy for preventing nonvertebral fractures. Teripratide is cost prohibitive and would only be in patients who fail a bisphosphonate or have severe osteoporosis and multiple fractures at baseline.
A 78 year old Asian female has a history of osteoporosis which has never been treated. You are asked to recommend appropriate treatment options for this patient who has a
history of hypertension, dyslipidemia, end stage renal disease (glomerular filtration rate 24 ml/min), recent venous thromboembolism, allergic rhinitis, and depression.
Below are the antiosteoporotic drugs and doses on formulary where you work.
I. zoledronic acid (ReclastTM) 5 mg intravenously once yearly
II. ibandronate (BonivaTM) 150 mg once monthly
III. raloxifene (EvistaTM) 60 mg once daily
IV. calcitonin (MiacalcinTM) 200 IU in alternating nostrils once daily
V. teriparatide (ForteoTM) 20 mcg subcutaneously once daily

Which of the following therapies would you include in your recommendations?
A. III only
B. I and II
C. IV and V
D. III, IV, and V
C. IV and V
BV has heard about the results from the WHI (Women’s Health Initiative) and would like for you to clarify the risks for her. The risk of what condition significantly decreased with hormone therapy (HT) in patients without a uterus taking dailyestrogen?
A. Breast cancer
B. Venous thromboembolism
C. Stroke
D. None of the above
D. None of the above. The only significant finding in the estrogen arm of the WHI was a reduction in hip fractures.
BV is a 52 year-old female who presents with complaints of hot flashes and insomnia. She has been without menses for 1.5 years. Laboratory values at your disposal include: TSH/T4 WNL (within normal limits), Basic metabolic panel WNL, CBC
WNL, FSH 80 pg/ml, t-score (-)1.2, TC 220 mg/dl, TG 190 mg/dl, HDL 58 mg/dl, LDL 124 mg/dl. Vital signs: HR 70 bpm, BP 122/70 mm Hg, T 99 C, RR 15, pain 0. She has a PMH of hypertension and surgical history of a hysterectomy. Medications:
acetaminophen 500 mg prn pain or HA and perindopril (Aceon®) 8 mg daily. She understands the risks and benefits of HT (hormone therapy) and would like to initiate HT for vasomotor symptoms (hot flashes). What initial HT regimen is most appropriate for BV?
A. Conjugated equine estrogen (CEE)(Premarin®) 0.625 mg daily
B. EstringTM vaginal ring, insert 1 vaginal ring (delivering 7.5 mcg
estradiol/24 hours) into the upper third of the vaginal vault. Keep in place
continuously for 3 months
C. Estradiol 0.05mg/24h, norethindrone acetate 0.14mg/24h
transdermal patch (CombiPatchTM), replace patch every 3-4 days
D. Estradiol 0.025 mg/24h, transdermal patch (Climara ® ), replace patch every 7 days
D. Estradiol 0.025 mg/24h, transdermal patch (Climara ® ), replace patch every 7 days

Patient does NOT have a uterus; thus, unopposed estrogen may be utilized and is associated with reduced risks as compared to E/P combination. Oral estrogens increase triglycerides and activate the RAS. Transdermal estrogens do not have these effects and are utilized in patients with controlled HTN, elevated triglycerides,
and at risk of VTE and cholelithiasis. Estring is only indicated for treatment of
vaginal symptoms due to minimal systemic absorption.
KJ is a healthy 21 year-old female who previously received a prescription for Necon 1/35 (1 mg norethindrone, 35 mcg ethinyl estradiol) from her doctor. She has been taking the Necon 1/35 for 1 month. She was advised of the risks and benefits of oral contraception and told to contact the office if she had any questions or problems. The patient is about to refill her prescription to begin her second month of therapy. She indicates that she had some breakthrough bleeding/spotting during this first month of taking the Necon 1/35 and it occurred on day 7 of her cycle.

Based on the above information, which of the following would be the most appropriate recommendation to this patient?
A. Recommend she keep taking the Necon 1/35 since the breakthrough bleeding often resolves within 3 months
B. Recommend she switch to a minipill such as Nor-QD (0.35 mg norethindrone)
C. Recommend she discontinue the Necon 1/35 and tell her that she should consider a barrier method of contraception
D. Recommend she switch to Demulen 1/35 (1 mg ethynodiol diacetate, 35 mcg ethinyl estradiol)
E. Recommend she switch to Ortho-Novum 1/50 (1 mg norethindrone, 50 mcg mestranol)
A. Recommend she keep taking the Necon 1/35 since the breakthrough bleeding often resolves within 3 months

S he’s been on this therapy for 1 month so spotting is likely to improve within first 3 months. Therefore, no need to change.
MT is a 20 year-old female who calls your pharmacy and indicates that she takes Yaz (drospirenone/ethinyl estradiol) for birth control and she’s been taking it for a year. She indicates that she is normally very compliant with her Yaz but that she was busy studying for finals the last couple of days and that she missed 3 consecutive doses. What is the most appropriate recommendation?
A. Forget about the pills she missed and just resume taking it today at 1 pill per day
B. Take 2 pills today then resume 1 pill daily tomorrow
C. Take 2 pills today, 2 pills tomorrow, then resume normal dose of 1 per day
D. Take 3 pills today then resume 1 per day tomorrow
E. Discontinue the Yaz for the rest of the pack and restart on a new Sunday
E. Discontinue the Yaz for the rest of the pack and restart on a new Sunday

If a patient misses 3 doses, then she d/c for rest of that pack and restart on a new Sunday
JP is a 42 year-old male initially diagnosed 2 years ago with Non-Hodgkin’s Lymphoma (NHL). He underwent 6 cycles of CHOP (cyclophosphamide, doxorubicin, vincristine, prednisone) chemotherapy without any complications. Two weeks ago, he reported a history of recent weight loss, decreased appetite, and diffused lymphadenopathy.
Subsequent workup confirmed recurrent NHL. He is now admitted to receive the first cycle of DHAP regimen:
Dexamethasone 40 mg PO daily on Days 1-4
Cisplatin 100 mg/m2 as continuous IV infusion on Day 1
Cytarabine 2 g/m2 IV q12h for 2 doses on Day 2
To be repeated q21-28d

What is the emetic risk associated with this (DHAP) chemo regimen?
A. High: greater than 90% risk
B. Moderate: 30-90% risk
C. Low: 10-30% risk
D. Minimal: less than 10% risk
E. None of the above
A. High: greater than 90% risk

A is the correct answer. According to the ASCO 2006 Antiemetic Guidelines, this cisplatin-containing chemotherapy is of high emetic risk. Greater than 90% of patients on this regimen would experience emesis without appropriate antiemetic therapy. Hence, Answers B, C, D & E are incorrect. Please note that Cisplatin belongs to the high emetic risk category whereas cytarabine > 1 g/m2 belongs to the moderate risk category. The chemo agent of the greatest emetic risk in the combination should be used to determine the most appropriate antiemetic regimen.
Which of the following is the most appropriate antiemetic regimen to prevent acute emesis on Day 1 for JP?
A. Dronabinol (Marinol) 50 mg PO
B. Granisetron (Kytril) 2 mg PO &amp; Dexamethasone (Decadron) 20 mg PO
C. Dolasetron (Anzemet) 100 mg PO &amp; Aprepitant (Emend) 125 mg PO
D. Ondansetron (Zofran) 24 mg PO &amp; Dexamethasone (Decadron) 12 mg PO &amp; Aprepitant (Emend) 125 mg PO
E. Ondansetron (Zofran) 24 mg PO
C. Dolasetron (Anzemet) 100 mg PO &amp; Aprepitant (Emend) 125 mg PO

C is the correct answer. A 3-drug combination of a 5-HT3 antagonist, steroid, &amp; aprepitant is recommended for the prevention of acute emesis for this chemo regimen of high emetic risk. Since JP is already on high dose Dexamethasone as a part of the DHAP chemo regimen, no additional steroid will be needed. Answers B &amp; D are incorrect since the additional Dexamethasone would provide too much steroid in this patient. Answers A &amp; E are incorrect because they are not as effective as the 3-drug antiemetic combination for chemo of high emetic risk. In addition, the dose of Dronabinol is too high.
Given Palonosetron (Aloxi) is not on the hospital formulary, which of the following is
the most appropriate antiemetic regimen to prevent acute &amp; delayed emesis on Day
2 for JP?
A. Granisetron (Kytril) 2 mg PO &amp; Dexamethasone (Decadron) 8 mg PO
B. Dolasetron (Anzemet) 100 mg PO &amp; Aprepitant (Emend) 80 mg PO
C. Aprepitant (Emend) 80 mg PO
D. Ondansetron (Zofran) 24 mg PO &amp; Aprepitant (Emend) 125 mg PO
E. Granisetron (Kytril) 2 mg PO &amp; Aprepitant (Emend) 80 mg PO &amp; Dexamethasone (Decadron) 8 mg PO
B. Dolasetron (Anzemet) 100 mg PO &amp; Aprepitant (Emend) 80 mg PO

B is the correct answer. On Day 2, JP is at risk of experiencing both acute emesis from high dose Cytarabine as well as delayed emesis from Cisplatin. The optimal antiemetic regimen should consists of a 5-HT3 antagonist (for acute emesis), Aprepitant &amp; Dexamethasone (for delayed emesis). For delay emesis, it is
appropriate to continue Aprepitant 80 mg PO on Day 2. Again, since JP is already on high dose Dexamethasone from DHAP, answers A &amp; E providing additional steroid are incorrect. Answer C is not optimal since Cytarabine 2 g/m2 given on Day 2 is of moderate emetic risk, a 5-HT3 antagonist should be included for the prevention of acute emesis on Day 2. Answer D is incorrect because the dose of Aprepitant on Day 2 is too high; it should be 80 mg PO.
Which of the following is the most appropriate regimen for breakthrough emesis for JP?
A. Ondansetron (Zofran) 8 mg ODT PO q4h prn
B. Scopolamine patch (Transderm Scop) 1.5 mg/24 h patch topically x 1
C. Droperidol 1.25-2.5 mg IV q6h prn
D. Diphenhydramine 12.5 mg PO/IV q4-6h prn
E. Prochlorperazine (Compazine) 10 mg PO/IV q4-6h prn
E. Prochlorperazine (Compazine) 10 mg PO/IV q4-6h prn

E is the correct answer. Since JP is to receive a 5-HT3 antagonist as premedication for acute emesis, Prochlorperazine is the most appropriate since the drug works differently by blocking dopamine receptors. The availability of iv injectables provides therapeutic alternative if JP cannot tolerate po. Answer A is incorrect because Ondansetron shares the same MOA of 5-HT3 receptor blockage and the dosing frequency is inappropriate. Answer B is incorrect because scopolamine patches have a delayed onset of action and do not work fast enough when breakthrough emesis occurs. Answer C is not ideal due to the FDA Black Box warning related to QT prolongation & torsades de pointes reported with Droperidol administration and the requirement of additional cardiac monitoring. Answer D is not the best answer because Diphenhydramine (an antihistamine) used alone at 12.5 mg low dose is not as potent and effective as compared to a dopamine receptor antagonist for chemo-induced emesis.
GA is a 65 year-old male recently diagnosed with stage III colorectal cancer. Patient has no other co-morbidities. Based on the available data, the best course of action is:
A. Infusional 5FU/Leucovorin shown to be better than bolus 5FU delivery
B. Folfox (5-FU, leucovorin, oxaliplatin); based on the mosaic trial that showed improved disease free survival.
C. Folfox (5-FU, leucovorin, oxaliplatin) + bevacizumab; based on the Tree1 and Tree2 studies which showed a 10% improvement in overall survival as compared to Folfox alone.
D. IFL; Irinotecan is a highly active agent in the treatment of colorectal cancer.
B. Folfox (5-FU, leucovorin, oxaliplatin); based on the mosaic trial that showed improved disease free survival.

Per NCCN guidelines, based on the Mosaic trial, Folfox is the initial regimen of choice. FU/leucovorin is not the initial choice unless there are mitigating /ircumstances. Bevacizumab has data for use in the metastatic setting, no current data for use in stage III disease. IFL is not indicated for treatment of early stage disease and has now been shown to be no better than 5FU/LV alone.
Risk factors for colorectal cancer include all of the following except
A. Obesity
B. Mutations in APC gene
C. History of polyps or inflammatory bowel disease
D. Consistent and constant use of aspirin
D. Consistent and constant use of aspirin

The use of aspirin might be associated with a lower incidence of colorectal cancer (extrapolated from the decreased development of adenomas), all other options are associated with a higher incidence of CRC.
AJ is 65 year old with a PSA (prostate specific antigen) of 2ng/ml. What is the most appropriate course of action?
A. Observation
B. Further work-up with TRUS
C. Radical Prostatectomy
D. Estramustine
A. Observation

TRUS is only recommended as a follow up for an abnormally elevated PSA.
Radical prostatectomy is not indicated as PSA is still within normal range.
Estramustine as a single agent does not have a role in the current care of prostate cancer patient.
A patient presents to your clinic with stage T1-2 prostate cancer without nodal involvement. Appropriate therapy at this time is:
A. Treatment with LHRH agonist
B. Combined treatment with LHRH agonist + antiandrogen
C. Radical prostatectomy
D. Treatment with surgery followed by FOLFOX (5-FU, leucovorin, oxaliplatin)
C. Radical prostatectomy

Radical prostatectomy in early stage disease has a >90% survival rate at 5 years. The other options are reserved for more advanced disease. FOLFOX is a regimen used in the treatment of colon cancer.
One of your friend’s father has been battling prostate cancer for a number of years
now. His last scan has shown metastatic disease of the hip. His testosterone level
was checked and came back elevated. He is currently on therapy with leuprolide and
flutamide. Patient suffers consistently with hip pain. Patient is still interested in
continuing treatment to prolong life. You appropriately recommend:
A. Since patient is progressing, the combined therapy with leuprolide and flutamide has failed. Would begin treatment with chemotherapy.
B. Since patient is progressing on combined therapy would withdraw the
antiandrogen (leuprolide).
C. Since patient is progressing on combined therapy would withdraw the
antiandrogen (flutamide).
D. Since patient has metastatic disease, he is now out of options. The best course of action would be to control his pain with narcotics and begin an IV bisphosphonate to reduce incidence of skeletal events.
C. Since patient is progressing on combined therapy would withdraw the
antiandrogen (flutamide).

Since patient disease is progressing with a rising testosterone level, patient is not a candidate for chemotherapy. The standard of care for patient progressing on CAB is to withdraw the antiandrogen (flutamide). Patient is not yet a candidate for palliative therapy.
CA is a new patient in your clinic who has disease progression (prostate cancer) despite testosterone levels <20ng/ml. He is very much interested in continuing therapy to increase his life expectancy. Considering his disease status, desire and your knowledge of clinical studies, you appropriately recommend:
A. Mitoxantrone 12mg/m2 IV q3wks + Prednisone 5mg PO BID, since it has shown to increase overall survival
B. Mitoxantrone 12mg/m2 IV q3wks + Prednisone 5mg PO BID, since it has shown to increase quality of life
C. Docetaxel 75mg/m2 IV q3wks + Prednisone 5mg PO BID, since it has shown to increase overall survival
D. Docetaxel 75mg/m2 IV q3wks + Prednisone 5mg PO BID, since it has shown to increase quality of life
C. Docetaxel 75mg/m2 IV q3wks + Prednisone 5mg PO BID, since it has shown to increase overall survival

Mitoxantrone + Prednisone has not been shown to increase survival. Docetaxel and Prednisone have been shown to increase overall survival. The patient, at this time, wants therapy that would increase overall survival and not quality of life.
JH is a 55 year-old patient who presented to his local doctor 2 weeks ago with
complaints of worsening cough with occasional blood-tinged sputum, dyspnea,
hoarseness, and a progressive weight loss (normally weighs 160 lbs). He had
recently been treated for a presumed bronchitis with clarithromycin for 7 days.
Additionally, he complained of a tingling sensation with weakness in the left arm. A
chest X-ray reveals a right peripheral lung mass, and a CT scan confirms a 2 x 3 cm
mass in his right lung and hilar lymph node involvement. A biopsy of his lung mass
is positive for small cell lung cancer, and he is diagnosed with Limited stage Small
Cell Lung Cancer.
Which of the following is the most appropriate initial course of treatment for JH?
A. Chemotherapy with single-agent topotecan (Hycamtin)
B. Chemotherapy with single-agent docetaxel (Taxotere)
C. Combination chemotherapy with etoposide (Toposar) + cisplatin
D. Combination chemotherapy with cisplatin/gemcitabine
C. Combination chemotherapy with etoposide (Toposar) + cisplatin

As JH has SCLC, chemotherapy is imperative. Treatment of choice is the
combination of etoposide + cisplatin, therefore “C” is the correct answer.
Topotecan is active in SCLC, but is generally considered treatment of choice
for relapsed SCLC. Docetaxel’s activity in SCLC is modest compared to
some of the other single agents, and is not often used to treat this disease.
Finally, the combination of cisplatin and gemcitabine is likely to have
activity, but presently there is no data to suggest it should replace
etoposide/cisplatin as first-line therapy.
Which of the following is TRUE regarding clinical trial results of epidermal growth factor (EGFR) tyrosine kinase inhibitors in the treatment of NSCLC (non-small cell lung cancer)?
A. Gefitinib (Iressa) is approved for all patients with advanced NSCLC previously treated with chemotherapy.
B. Gefitinib (Iressa) is associated with benefit in patients who never smoked and who are of Asian race.
C. Single-agent therapy with erlotinib (Tarceva) is approved for first-line treatment of NSCLC
D. Erlotinib (Tarceva) is ineffective in patients with detectable EGFR mutations.
B. Gefitinib (Iressa) is associated with benefit in patients who never smoked and who are of Asian race.

A is incorrect, as the initial approval for gefitinib was only for patients with
refractory disease. When the final analysis of the ISEL study revealed no
survival benefit, gefitinib’s approval was limited to those patients already on
therapy with a response. Presently, no new patients may be started on this
therapy. Presently, erlotinib is indicated only in patients who have failed
previous therapy, so “C” is incorrect as well. Lastly, the EGFR inhibitors,
such as erlotinib and gefitinib appear to be most effective in patients whose
tumors express mutated EGFR. Since analysis of EGFR mutation status is
technically complicated, surrogate factors have been identified, including
never-smoker status, female sex, and Asian heritage.
G is a 58 year-old woman who was diagnosed 1 year ago with metastatic breast cancer with primarily bone involvement. Her primary tumor was ER positive, PR positive, and HER2 positive. She received initial therapy with doxorubicin and docetaxel (AC) in combination with trastuzumab (Herceptin). While she completed her chemotherapy 3 months ago, she has continued to receive trastuzumab every 3 weeks. She has been on anastrazole (Arimidex) since completing her chemotherapy. She now presents with increased hip pain, and is found to have new involvement in her left hip and her spine.
Which of the following is the most appropriate therapy for SG’s recurrent
metastatic breast cancer?
A. Continue trastuzumab, begin paclitaxel and bevacizumab (Avastin), and change anastrazole to exemestane (Aromasin)
B. Continue trastuzumab, begin gemcitabine (Gemzar), and change anastrazole to exemestane Aromasin)
C. Discontinue trastuzumab, begin lapatinib (Tykerb) and capecitabine (Xeloda), and change anastrazole to exemestane (Aromasin).
D. Discontinue trastuzumab, begin lapatinib (Tykerb) and capecitabine (Xeloda), and discontinue anastrazole.
C. Discontinue trastuzumab, begin lapatinib (Tykerb) and capecitabine (Xeloda), and change anastrazole to exemestane (Aromasin).

Since she has progressed on trastuzumab, further trastuzumab is not likely
to provide benefit, even if combined with different chemotherapy agents,
which makes choices “A” and “B” incorrect. The combination of lapatinib
and capecitabine has been approved for patients with recurrent HER2-
positive metastatic breast cancer. As her tumor is hormone receptor positive,
she should continue to receive some form of hormonal therapy; a switch in
therapy from anastrazole to another agent is appropriate. Bevacizumab has
not been shown to be effective in the 2nd or 3rd line treatment of metastatic
breast cancer, so should not be used.
KJ is a 64 year-old postmenopausal African-American woman with newly diagnosed breast cancer, stage IIIA (T2, N2, M0). Her tumor is positive for both estrogen and progesterone receptor expression, and is strongly positive for HER2 overexpression by immunohistochemistry (IHC 3+). She is status-post mastectomy, and presents for adjuvant therapy.
Which of the following is the most appropriate option for KJ?
A. Chemotherapy plus trastuzumab (Herceptin), followed by hormonal therapy
B. Chemotherapy plus trastuzumab (Herceptin)
C. Chemotherapy followed by hormonal therapy
D. Chemotherapy followed by radiation therapy
A. Chemotherapy plus trastuzumab (Herceptin), followed by hormonal therapy

KJ should receive chemotherapy, given her more advanced disease stage and
positive lymph node status. Because of her HER2 overexpression, she is at
significant risk of relapse; data from a number of adjuvant supports the use
of adjuvant trastuzumab in this patient population. Lastly, as her tumor is
hormone-receptor positive, she should receive hormonal therapy, preferably
with an aromatase inhibitor. Only option “A” offers all three treatment
approaches.
TJ is a 35 year-old premenopausal woman presenting with newly diagnosed left breast cancer. Two months ago, she noticed that her left nipple was retracted and discovered a lump during a self-exam. A diagnostic mammogram revealed a 2.5 cm speculated mass in her left breast without calcifications. She underwent a lumpectomy and lymph node section, which revealed involvement in 2 of 14 nodes. Final pathology confirmed a 2.5 cm primary mass that was negative for ER or PR expression and negative for HER2 overexpression by FISH. She is diagnosed with a Stage IIB breast cancer.
Which of the following is the most appropriate therapy for TJ?
A. 4 cycles of doxorubicin + cyclophosphamide (AC) followed by 12 weeks of paclitaxel (P) + 1 year of trastuzumab (Herceptin)
B. 4 cycles of doxorubicin + cyclophosphamide (AC) followed by 12 weeks of paclitaxel (P) + aromatase inhibitor
C. 4 cycles of doxorubicin + cyclophosphamide (AC) followed by 12 weeks of paclitaxel (P) + bevacizumab (Avastin)
D. 4 cycles of doxorubicin + cyclophosphamide (AC) followed by 12 weeks of paclitaxel (P)
D. 4 cycles of doxorubicin + cyclophosphamide (AC) followed by 12 weeks of paclitaxel (P)

\As TJ’s tumor was negative for hormone receptors, hormonal therapy is not
expected to provide benefit. Similarly, she should not receive trastuzumab, as
her tumor was negative for HER2 overexpression. At present, bevacizumab
has no role in the treatment of stage II breast cancer; it is currently approved
in combination with paclitaxel alone as first-line treatment for metastatic
(Stage IV) breast cancer. Therefore, she should be treated with
chemotherapy as in “D”.
Which of the following chemotherapy agents is NOT associated with the
development of peripheral neuropathy?
A. Vincristine
B. Ixabepilone (Ixempra)
C. Cyclophosphamide
D. Paclitaxel
C. Cyclophosphamide

The vinca alkaloids (vincristine), the taxanes (paclitaxel), and the new class
of drugs the epothilones (ixabepilone) are all associated with the development
of significant neuropathy, which may limit their usefulness in some patient
populations.
AP is a 45 year-old female with breast cancer, who recently began treatment with doxorubicin and cyclophosphamide (AC) chemotherapy. She refused to have an Infusa-Port placed, and received her chemotherapy via a peripheral vein. During her 2nd cycle of chemotherapy, the needle became dislodged during the doxorubicin infusion, and about 10 ml of drug infiltrated into her arm tissue. Which of the following approaches is best for managing a doxorubicin extravasation?
A. Inject area with hylauronidase
B. Inject area with 1/6M thiosulfate
C. Apply heat
D. Apply cold
D. Apply cold

Doxorubicin can cause very severe tissue damage, potentially leading to
tissue necrosis. Application of cold to the area will result in vasoconstriction,
minimizing exposure to the drug.
Which of the following statements regarding monoclonal antibodies is FALSE?
A. Monoclonal antibodies are more selective for tumor cells than chemotherapy
B. Monoclonal antibodies cause hemorrhagic cystitis to the same degree as cyclophosphamide chemotherapy
C. Monoclonal antibodies have a long half-life, and therefore require intermittent administration.
D. Monoclonal antibodies have a limited ability to penetrate into tumors.
B. Monoclonal antibodies cause hemorrhagic cystitis to the same degree as cyclophosphamide

Advantages associated with monoclonal antibody therapy include better
selectivity to cancer cells and therefore less toxicity to normal cells, therefore
they do not cause severe organ toxicity. They do have fairly long half-lives, so
generally require dosing no more often than once a week. They do not
penetrate well into bulky tumors. Therefore, “B” is the correct answer.
Characteristics of benign tumors include all of the following EXCEPT:
A. They are usually localized and/or encapsulated
B. They are genetically stable
C. They demonstrate abnormal growth (anaplasia, dysplasia)
D. Once removed, they rarely recur
C. They demonstrate abnormal growth (anaplasia, dysplasia)

Abnormal growth is a characteristic of a malignant tumor. Benign tumors
tend to demonstrate more orderly growth.
Mr YI is a 56-year-old male undergoing induction treatment for leukemia. On admission he was febrile, with a maximum temperature of 39oC (102.2 oF). Shortly thereafter his
blood pressure fell to 77/50 mmHg, he was tachycardic and his urine output fell significantly.

Which of the following most accurately describes the necessary treatment intervention?
A. Initiate broad-spectrum antibiotics with a 3rd/4th generation cephalosporin and send the patient for a chest X-ray for follow-up.
B. Initiate broad-spectrum antibiotics with a 3rd/4th generation cephalosporin in combination with a glycopeptides.
C. Initiate broad-spectrum antibiotics with a 3rd/4th generation cephalosporin in combination with a glycopeptides and an aminoglycoside and send the patient for a chest x-ray for follow-up.
D. Initiate broad-spectrum antibiotics with a glycopeptides and send the patient for a chest x-ray for follow-up.
C. Initiate broad-spectrum antibiotics with a 3rd/4th generation cephalosporin in combination with a glycopeptides and an aminoglycoside and send the patient for a chest x-ray for follow-up. The reason it is correct is that the patient is septic (with the presenting signs and symptoms of tachycardia, hypotension and falling urine output). In this setting one should treat as broadly as possible with gram positive, gram-negative coverage and the patient should have follow-up scans for a fever spike. The other options are therefore incorrect because they are either monotherapy or 2-drug therapy.
Mr. YI’s temperature decreases over the next 48 hours and he stabilizes in the intensive care unit. He remains afebrile for several days, and then spikes another temperature to 40.3oC (104.5 oF). He is clinically stable, but complains of pleuritic chest pain on deep inspiration. He has a non-productive cough. A CT scan of the chest is performed and is positive for an air crescent sign.

What is the most likely cause of his clinical presentation?
A. Aspergillus pneumonia
B. Candida pneumonia
C. Nocardia pneumonia
D. Haemophilus influenza pneumonia
A. Aspergillus pneumonia

The hallmark symptoms for invasive aspergillosis are fever; cough, pleuritic chest pain and the CT findings strongly suggest aspergillosis. Candida pneumonia is a rare entity and unlikely with this presentation. Nocardia
can cause pneumonia in this patient population but would typically have nodules on CT scan not an air crescent sign, nor be associated with pleuritic chest pain. Haemophilus influenza pneumonia would not cause this clinical and radiologic picture.
Patients can be categorized into three main risk groups when discussing febrile
neutropenia. Which of the following patients is not in the high-risk category?
A. Breast cancer receiving adjuvant chemotherapy
B. Allogeneic hematopoietic stem cell transplant candidate
C. Acute myeloid leukemia undergoing induction chemotherapy
D. Acute lymphoblastic leukemia in the elderly with significant comorbidities
A. Breast cancer receiving adjuvant chemotherapy

High risk patients are the following: prolonged neutropenia (>= 14 days), hematologic malignancy or allogeneic bone marrow
transplantation, substantial comorbidity, clinically unstable, slow response to initial therapy. Therefore “a” is not within this category and is in the low-risk category.
Which of the following most accurately describes the initial treatment of choice for invasive aspergillosis?
A. Caspofungin (Cancidas) 70 mg IV loading dose x 1 dose followed by 50 mg IV
daily.
B. Voriconazole (Vfend) 6 mg/kg IV every 12 hours x 2 doses, followed by 4
mg/kg IV twice daily thereafter
C. Fluconazole (Diflucan) 400 mg PO/IV daily
D. Itraconazole (Sporanox) 400 mg IV twice daily
B. Voriconazole (Vfend) 6 mg/kg IV every 12 hours x 2 doses, followed by 4
mg/kg IV twice daily thereafter

Caspofungin has activity against Aspergillosis but has not
been evaluated in the front-line management of Aspergillosis. Voriconazole was evaluated in the Herbrecht study, as is the standard of care. Fluconazole has no activity against Aspergillus spp. Itraconazole has activity against Aspergillus, but the dose is not correct.
The following are known toxicities of amphotericin B EXCEPT?
A. Renal tubular acidosis
B. Visual disturbances
C. Infusion related adverse events
D. Phlebitis
B. Visual disturbances

A, C, and D are all recognized adverse events associated with amphotericin B therapy. Visual disturbances are associated with voriconazole therapy, not amphotericin B therapy.
Which of the following patients with acute lymphoblastic leukemia is potentially the most curable?
A. A 64-year-old male with piladelphia chromosome positive acute lymphoblastic leukemia
B. A 45-year-old male with acute lymphoblastic leukemia with a WBC count of 65 x 109/L at presentation
C. A 5-year-old child with acute lymphoblastic leukemia
D. A 25 year old female with acute lymphoblastic leukemia with a WBC count of 6 x 109/L and the presence of a mediastinal mass at presentation
C. A 5-year-old child with acute lymphoblastic leukemia

ALL is a curable disease in pediatrics. Philadelphia
chromosome ALL is associated with a poor prognosis, particularly in the elderly
and as such A is incorrect. Patients who present with high WBC counts are poor risk, so less curable. A mediastinal mass is a poor prognostic factor.
Mrs MC, a 43-year-old female presented to her local doctor in March 2005 with mild anemia and leukocytosis. An extensive workup was unremarkable. The bone marrow biopsy was 30% cellular with normal hematopoiesis. This was thought to be due to iron deficiency anemia, and she was started on iron replacement. She represented in July 2006, with pancytopenia. Her WBC count was 0.6 x 109/L, Hb 4.6g/L, and platelets 50 x
109/L. A bone marrow biopsy at that time was hypercellular with many immature cells (blasts). Flow cytometry (immunophenotyping) was positive for CD13, CD33, CD34, HLA-DR. Cytochemical staining was myeloperoxidase positive. On physical exam she has petechiae (small purple spots caused by small hemorrhages) and complains of occasional epistaxis (nosebleeds). She has no B-symptoms.

Based on the signs, symptoms, and laboratory tests described, Mrs MC most likely has which type of malignancy?
A. Chronic lymphocytic leukemia
B. Acute lymphocytic leukemia
C. Non-Hodgkin’s lymphoma
D. Acute myeloid leukemia
D. Acute myeloid leukemia

Her flow cytometry markers that are positive are indicative of AML. She is also pancytopenic which is common in this malignancy and unlikely in NHL and CLL unless there is complete marrow replacement. She does not have any B-symptoms – this makes A and C unlikely. ALL would have different flow cytometry markers.
Based on the case of Mrs MC, the best induction chemotherapy regimen for this patient would include:
A. Cytarabine in combination with an anthracycline
B. Fludarabine in combination with cyclophosphamide
C. Methotrexate, 6-mercaptopurine, with or without prednisone
D. Cyclophosphamide, doxorubicin, vincristine, prednisone, rituximab
A. Cytarabine in combination with an anthracycline

This is the treatment for AML. B would be used for CLL. C would be used for ALL maintenance and D would be used for NHL.
Treatment of CLL is generally initiated when patients have:
A. Stage 0 disease
B. Stage I disease
C. Stage II disease
D. Stage III-IV disease
D. Stage III-IV disease

Treatment of CLL is only initiated when the patient is symptomatic which is Stage III-IV disease.
MS, a 63-year-old woman, presents with a history of acute low back pain. She had menopause at 44 years of age but never received postmenopausal hormone-replacement therapy. She denies a history of fragility fracture. Her mother sustained a hip fracture at 70 years of age. Lumbar-spine films reveal a new vertebral fracture. Dual-energy x-ray absorptiometry of the hip shows a bone mineral density T score of –1.3.

Which of the following interventions would be most appropriate?
A. Alendronate (FosamaxTM) 10 mg weekly
B. Risedronate (ActonelTM) 35 mg weekly
C. Calcitonin (MiacalcinTM) 200 IU in alternating nostrils daily
D. Recheck bone mineral density in 5 years
B. Risedronate (ActonelTM) 35 mg weekly

A bisphosphonate is clearly indicated in this patient given the history of fracture
and presence of multiple risk factors. Even though her BMD is not impressive, the presence of multiple risk factors warrants pharmacological intervention now (ruling out D). In the absence of contraindications, a bisphosphonate is considered first line (ruling out C). Calcitonin is not a first line agent due to limited efficacy for preventing nonvertebral fractures. The dose of alendronate in answer A is incorrect. Thus, risedronate 35 mg weekly is the best option.
JK was started on ibandronate (BonivaTM) 150 mg monthly approximately 6 months ago following a vertebral fracture. Her BMD was determined to be -2.4. JK has a medical history notable for hypertension (156/78 mmHg), gastroesophageal reflux disease, depression (diagnosed February 2002), and venous thromboembolism (January 2007). She complains of significant abdominal pain and musculoskeletal aches that have not resolved despite the fact that she has strictly adhered to the counseling you provided around appropriate administration of bisphosphonates. Which of the treatments below could be considered in managing her osteoporosis?
A. Teriparatide (ForteoTM)
B. Calcitonin
C. Raloxifene (EvistaTM)
D. A and B
E. All of the above
D. A and B
GI intolerance and musculoskeletal pain are reported much more frequently with bisphosphonates in clinical practice versus the clinical trial experience. JK did not tolerate ibandronate even when given as infrequently as monthly. Raloxifene should not be considered in this case due to the history of recent venous thromboembolism. Both teriparatide and calcitonin are options for decreasing this patient’s risk.
MP is a 50 year-old White Female who presents to your family medicine clinic complaining of hot flashes and irregular menses. She has discussed these issues with a few of her friends and they have given her advice on lifestyle modifications to reduce vasomotor symptoms. What lifestyle modification is NOT associated with reducing vasomotor symptoms?
A. Regular exercise
B. Smoking cessation
C. Meditation
D. Drinking a glass of wine
D. Drinking a glass of wine

Alcohol may worsen vasomotor symptoms. All others are recommendations to reduce the frequency of hot flushes/flashes.
MP has heard about the results from the WHI (Women’s Health Initiative) study and would like for you to clarify the risks for her. Risk of what condition is NOT increased with hormone therapy (HT) in patients with an intact uterus taking daily estrogen and progestogen?
A. Colorectal cancer
B. Venous thromboembolism
C. Stroke
D. Breast cancer
A. Colorectal cancer

The risk of colon cancer was decreased in patients in the CEE/MPA arm of the
WHI. All others listed were increased per slide 31 and 32
MP attempted a trial of black cohosh with no resolution of symptoms. She understands the risks and the benefits of hormone therapy (HT) and would like to initiate HT for vasomotor symptoms (hot flushes) and vaginal dryness. Currently, she is at low risk for complications associated with HT. Of note, MP is with uterus. What initial HT regimen is most appropriate for MP?
A. Conjugated equine estrogen (CEE) (Premarin®) 0.625 mg daily
B. EstringTM (estradiol) vaginal ring, insert 1 vaginal ring (delivering 7.5 mcg estradiol/24 hours) into the upper third of the vaginal vault. Keep in place continuously for 3 months
C. CEE (conjugated equine estrogen)/MPA (medroxyprogesterone acetate) 0.3/1.5 mg (Prempro®), 1 tablet daily
D. Venlafaxine (Effexor®) 75 mg, 1 tablet daily
C. CEE (conjugated equine estrogen)/MPA (medroxyprogesterone acetate) 0.3/1.5 mg (Prempro®), 1 tablet daily

Patient has a uterus; thus, unopposed estrogen increases the risk of endometrial hyperplasia and cancer. Estring is only indicated for treatment of vaginal symptoms. Venlafaxine is not as effective at HT and does not treat vaginal sx.
KJ is a healthy 21 year-old female who previously received a prescription for Ortho-Novum 1/35 (1 mg norethindrone, 35 mcg ethinyl estradiol) from her doctor. She has been taking the Ortho-Novum 1/35 for 5 months now. She was advised of the risks and benefits of oral contraception and told to contact the office if she had any questions or problems. The patient completes 5 months of therapy and presents to your pharmacy with the prescription. She indicates that she had some breakthrough bleeding/spotting each month since starting the Ortho Novum 1/35 and it occurs on day 8 of her cycle.

Based on the above information, which of the following would be the most appropriate recommendation to this patient?
A. Recommend she keep taking the Ortho-Novum 1/35 since the bleeding often gets better with time
B. Recommend she switch to a minipill such as Micronor (0.35 mg norethindrone)
C. Recommend she discontinue the Ortho-Novum 1/35 and tell her that she should consider a barrier method of contraception
D. Recommend she switch to Demulen 1/35 (1 mg ethynodiol diacetate, 35 mcg ethinyl estradiol)
E. Recommend she switch to Ortho-Novum 1/50 (1 mg norethindrone, 50 mcg mestranol)
D. Recommend she switch to Demulen 1/35 (1 mg ethynodiol diacetate, 35 mcg ethinyl estradiol)

Since she is spotting on day 8 (early/mid cycle) that indicates an estrogen deficiency. She’s been on this therapy for more than 3 months so spotting is not likely to improve at this point. Therefore, need to change to an agent with a progestin that has more estrogenic activity. That would be Demulen.
MT is a 20 year-old female who calls your pharmacy and indicates that she takes Yasmin (drospirenone/ethinyl estradiol) for birth control and she’s been taking it for 4 months. She indicates that she is normally very compliant with her Yasmin but that
she was busy studying for finals the last couple of days and that she missed 2
consecutive doses. What is the most appropriate recommendation?

A. Forget about the pills she missed and just resume taking it today at 1 pill per day
B. Take 2 pills today then resume 1 pill daily tomorrow
C. Take 2 pills today, 2 pills tomorrow, then resume normal dose of 1 per day
D. Take 3 pills today then resume 1 per day tomorrow
E. Discontinue the Yasmin for the rest of the pack and restart on a new Sunday
C. Take 2 pills today, 2 pills tomorrow, then resume normal dose of 1 per day

If a patient misses 2 doses, then she should take 2 per day for 2 days then resume normal dose.
CJ is a 40 year-old Male with a recent diagnosis of NHL (Non Hodgkins Lymphoma). He is an entrepreneur with a fast-paced lifestyle. He is very upset about the diagnosis as he is told by his oncologist that he has to slow down and take care of his health. He is in the clinic today to receive CHOP therapy (Cyclophosphamide 750 mg/m2 iv, Doxorubicin 50 mg/m2 iv, and Vincristine 2 mg all given on Day 1, and Prednisone 100 mg po daily on Days 1-5) q 21 days. CJ is instructed to take his Prednisone as a part of his chemotherapy treatment in the morning before he comes into the clinic.

Which of the following is the most appropriate antiemetic regimen to prevent acute emesis in CJ?
A. Prochlorperazine (Compazine) 10 mg iv
B. Granisetron (Kytril) 1 mg iv &amp; Dexamethasone (Decadron) 20 mg iv
C. Ondansetron (Zofran) 24 mg po &amp; Aprepitant 125 mg po
D. Dronabinol (Marinol) 25 mg po
E. Lorazepam (Ativan) 1 mg po
C. Ondansetron (Zofran) 24 mg po &amp; Aprepitant 125 mg po

C is the correct answer. According to the ASCO 2006 Antiemetic Guidelines, this cyclophosphamide-containing chemotherapy is of moderate emetic risk. A 3-drug combination of a 5HT3 antagonist, steroid, &amp; aprepitant is recommended for the prevention of acute emesis. Since this patient is already on Prednisone as a part of CHOP, no additional steroid will be needed. Answers A, B, D, &amp; E are incorrect because these combinations (with Prednisone) are not as potent as the 3-drug antiemetic combination. In addition, Answer B with the use of Dexamethasone provides too much steroid in this patient. Answer D is incorrect because Dronabinol needs to be given starting the day before chemotherapy, and the dose is too high. Answer E is incorrect because Lorazepam is for anticipatory not acute emesis.
CJ experiences severe nausea/vomiting 2 days after his first cycle of chemotherapy. What type(s) of emesis will he be likely to experience with subsequent cycles of chemotherapy?
I. Acute
II. Delayed
III. Anticipatory
IV. Breakthrough

A. II only
B. III only
C. I and II
D. I, II., and III.
E. I, II, III, and IV.
E. I, II, III, and IV.

E is the correct answer. Due to prior poor emetic control, CJ is at risk for
anticipatory emesis in addition to acute, delayed, and breakthrough n/v which are
common with this chemo regimen.
SL is a 46 year-old Female with a diagnosis of non-small cell lung cancer (NSCLC). She has been on Paclitaxel 175 mg/m2 and Carboplatin AUC 6 both administered iv on Day 1 and repeated q 21 days. She tolerated the first cycle well and is in the clinic today to receive her second cycle. She has been premedicated with Palonosetron (Aloxi) 0.25 mg and Dexamethasone (Decadron) 8 mg both given iv, 30 min before chemotherapy. Fifteen minutes into her Carboplatin infusion, she starts to have some nausea and retching.

Which type of nausea/vomiting is she currently experiencing?
I. Acute
II. Delayed
III. Anticipatory
IV. Breakthrough

A. II only
B. I and II
C. I, II, and III.
D. III only
E. IV only
E. IV only

E is the correct answer. She is experiencing breakthrough emesis despite of prophylactic medications for acute emesis. Answers A, B &amp; C are incorrect since SL’s emesis occurs within the 1st 24 hours of chemo administration rather than 24 hours after chemo, it is not delayed emesis. Answer D is incorrect because she did well with good emetic control during the 1st cycle of chemo. Her emesis occurs after (rather than before) chemo is given, therefore it is not anticipatory emesis.
Which one of the following regimens is the most appropriate to treat her current nausea?

A. Ondansetron (Zofran) 8 mg ODT po
B. Aprepitant (Emend) 125 mg po
C. Prochlorperazine (Compazine) 10 mg iv
D. Dexamethasone (Decadron) 20 mg po
E. Scopolamine patch (Transderm Scop) 1.5 mg/24 h patch topically
C. Prochlorperazine (Compazine) 10 mg iv

C is the correct answer. Prochlorperazine and iv are the most appropriate since the drug works differently by blocking dopamine receptors and can be given iv for breakthrough emesis in SL who is already nauseated. Since she has received a 5HT3 antagonist and steroid as premedication, Answers (A) ondansetron & (D) dexamethasone that have the same MOA will not provide a substantial clinical benefit, but potentially increase costs and overlapping toxicity. Answer (B) is incorrect because Aprepitant is best used preventively before chemo is given. Also, SL is already nauseated and may not be able to take po. Answer (E) is incorrect because Scopolamine patches have delayed onset of action and are undesirable for
immediate control of breakthrough emesis.
JH is a 55 year-old patient who presented to his local physician 2 weeks ago with complaints of cough, dyspnea, hoarseness, and chest pain. A chest X-ray reveals a right
peripheral lung mass, and a CT scan confirms a 3 x 4 cm mass in his right lung and hilar lymph node involvement. A biopsy of his lung mass is positive for squamous cell carcinoma, and he is diagnosed with non-small cell lung cancer, Stage IIA (T2, N1, M0).

Which of the following is the most appropriate course of treatment for JH?
A. surgical resection alone
B. surgical resection followed by adjuvant chemotherapy
C. combination chemotherapy + radiation therapy
D. combination chemotherapy + bevacizumab (Avastin)
B. surgical resection followed by adjuvant chemotherapy

As JH has lymph node involvement, surgical resection alone is insufficient,
and he will need post-operative therapy, which makes answer “A” incorrect.
C and D are both incorrect, as they don’t include surgical resection. These
approaches would be indicated for patients with unresectable stage III or
stage IV disease, in whom surgery is not an option. As JH has squamous cell
histology, bevacizumab is contraindicated due to increased risk of
pulmonary hemorrhage.
As his disease is confined to the one lung mass and regional nodes, resection
of the lung mass followed by systemic adjuvant therapy offers him the best
chance for long-term disease control.
BG is a 62 year-old female with newly diagnosed stage IIIB adenocarcinoma of the lung (NSCLC – non-small cell lung cancer). Her past medical history is significant for oxygen-dependent COPD, insulin-dependent diabetes, and peripheral neuropathies. She is referred for systemic chemotherapy.

Which of the following is the most appropriate regimen for BG?
A. Paclitaxel 175 mg/m2 IV day 1 and cisplatin 75 mg/m2 IV day 1, repeat Q 3 weeks
B. Paclitaxel 175 mg/m2 IV day 1 and carboplatin AUC 5, repeat Q 3 weeks
C. Docetaxel 75 mg/m2 IV day 1 and cisplatin 75 mg/m2 IV day 1, repeat Q 3 weeks
D. Gemcitabine (Gemzar) 1250 mg/m2 IV day 1 and 8, and cisplatin 75 mg/m2 IV day 1, repeat Q 3 weeks
D. Gemcitabine (Gemzar) 1250 mg/m2 IV day 1 and 8, and cisplatin 75 mg/m2 IV day 1, repeat Q 3 weeks

In the ECOG 1594 study, all 4 regimens provided similar efficacy (similar
response rates, similar time to progression, similar survival), although the paclitaxel/carboplatin combination appears to have less overall toxicity than the other three regimens. The taxanes, particularly paclitaxel, can cause significant neuropathies. Because of BG’s preexisting neuropathies, she is unlikely to tolerate taxane therapy very well. Therefore, the gemcitabine-cisplatin combination is the best choice in this particular patient.
JP is a 67 year-old postmenopausal female with a newly diagnosed right breast cancer, stage IIB (T2, N1, M0). Core biopsies of the breast mass indicate an invasive ductal carcinoma, ER positive, PR positive, HER2 neg by FISH. Staging studies were negative for metastases. She is s/p mastectomy, and on axillary lymph node dissection was noted to have 4 involved nodes. She now presents for adjuvant therapy.
Which of the following is the most appropriate therapy for JP?
A. Adjuvant anastrazole (Arimidex) for 5 years
B. Adjuvant chemotherapy with doxorubicin + cyclophosphamide x 4 cycles followed by paclitaxel x 4 cycles (AC x 4 T x 4)
C. Adjuvant radiation therapy and tamoxifen (Nolvadex) x 5 years
D. Adjuvant chemotherapy with doxorubicin + cyclophosphamide x 4 cycles followed by paclitaxel x 4 cycles (AC x 4 T x 4 cycles) followed by anastrazole (Arimidex) x 5 years
D. Adjuvant chemotherapy with doxorubicin + cyclophosphamide x 4 cycles followed by paclitaxel x 4 cycles (AC x 4 T x 4 cycles) followed by anastrazole (Arimidex) x 5 years

As she has stage IIB disease with lymph node involvement, she should get
adjuvant chemotherapy. As her primary tumor was hormone receptor positive, she should also get adjuvant hormonal therapy. Only answer “D” includes both chemotherapy and hormonal therapy. While adjuvant radiation should be administered following lumpectomy, it offers no benefit following mastectomy. Adjuvant hormonal therapy alone might be appropriate if she had stage I or stage IIA disease without lymph node involvement.
KJ is a 66 year-old postmenopausal female with newly diagnosed bone metastases. She was originally diagnosed with stage IIB breast cancer three years ago. Her original tumor was ER/PR positive, HER2 negative. She underwent a mastectomy followed byadjuvant chemotherapy with AC (cyclophosphamide/doxorubicin) x 4 cycles followed by paclitaxel x 4 cycles. After her chemotherapy was completed, she was started on tamoxifen, which she has been taking for about 2.5 years. She presented a week ago with new pain in her lower back that is relieved with OTC analgesics. A bone scan revealed
suspicious areas of uptake in her lumbar spine. Re-staging studies were negative for other areas of metastasis.
Which of the following is the most appropriate option for KJ?
A. Lapatinib (Tykerb) 1250 mg PO daily
B. Anastrazole (Arimidex) 1 mg PO daily
C. Docetaxel (Taxotere) 100 mg/m2 IV Q 3 weeks
D. Toremifene (Fareston) 60 mg PO daily
B. Anastrazole (Arimidex) 1 mg PO daily

She has newly diagnosed bone metastases occurring while on tamoxifen
therapy. According to the NCCN algorithm (slide #75 from the part 2
lecture), she is a candidate for another hormone therapy. As she is
postmenopausal, she is an excellent candidate for an aromatase inhibitor
(anastrazole, letrozole, exemestane). As she progressed while on tamoxifen,
her tumor is likely to be resistant to other estrogen receptor modulators such
as toremifene. Chemotherapy would not be the first choice in this patient
population, as hormonal therapy is very effective and well-tolerated.
Lapatinib is indicated in combination with capecitabine in patients whose
tumors are HER2 positive and who have failed prior therapy with a taxane,
anthracycline, and trastuzumab. As KJ’s original tumor was HER2 negative,
it is not appropriate for her.
TJ is a 35 year-old premenopausal woman presenting with newly diagnosed left breast cancer. She noticed a lump in her breast some time ago, but neglected to have it checked out. She now presents with dimpling of the skin of the breast, erythema, and palpable lymph nodes in the left axilla and supraclavicular areas. A biopsy is the breast mass reveals invasive ductal carcinoma, ER negative, PR negative, HER2 positive by FISH. Staging studies confirm axillary and supraclavicular lymph node involvement, the presence of bone metastases, and a lung lesion that is concerning for pulmonary metastases.

Which of the following is the most appropriate therapy for TJ?
A. Paclitaxel 175 mg/m2 IV day 1 + carboplatin AUC 6 IV day 1 + trastuzumab (Herceptin) 4 mg/kg loading dose followed by 2 mg/kg IV Q week
B. Paclitaxel 175 mg/m2 IV day 1 + carboplatin AUC 6 IV day 1
C. Lapatinib (Tykerb) 1250 mg PO daily + capecitabine (Xeloda) 2000 mg/m2 PO daily x 14 days, cycle Q 3 weeks
D. Bevacizumab (Avastin) 10 mg/kg IV Q 2 weeks + trastuzumab (Herceptin) 4 mg/kg loading dose followed by 2 mg/kg IV Q week
A. Paclitaxel 175 mg/m2 IV day 1 + carboplatin AUC 6 IV day 1 + trastuzumab (Herceptin) 4 mg/kg loading dose followed by 2 mg/kg IV Q week

TJ has metastatic breast cancer. Her tumor was ER neg, PR neg and HER2
positive. Trastuzumab added to chemotherapy has been shown to improve survival compared with chemotherapy alone, therefore answer “A” is the most appropriate. Lapatinib + capecitabine is indicated for patients whose tumors are HER2 positive and who have failed prior therapy with a taxane, anthracycline, and trastuzumab and thus is not appropriate for 1st line
therapy for metastatic disease. The combination of bevacizumab and
trastuzumab is still considered experimental and is not 1st line treatment for metastatic disease.
BS is a 63 year-old female who presented to her local doctor with recent bruising. She is found to have a high white blood cell count (WBC) and the presence of blasts is noted in her differential. She was transferred to the hospital for further evaluation and subsequent therapy. A bone marrow biopsy reveals 90% myeloblasts, and she is diagnosed with acute myelogenous leukemia, FAB M1. Her WBC is 24,000/mm3, and the differential reveals: neutrophils 2%, lymphocytes 17%, monocytes 0%, eosinophils 0%, bands 1%, and blasts 80%. The oncologist decides to begin induction chemotherapy with “7 + 3”, which consists of Cytarabine 100 mg/m2 IV continuous infusion x 7 days and idarubicin 12 mg/m2 IV daily x 3 days. She is 64.5 inches tall (163.8 cm) and weighs 162 pounds (73.6 kg).

Calculate BS’s absolute neutrophil count (ANC):
A. 240/mm3
B. 480/mm3
C. 720/mm3
D. 4,080/mm3
E. 19,200/mm3
C. 720/mm3

The ANC is calculated by multiplying the WBC by the percentage of neutrophils and bands, which is 3% in this example. 24,000 x 0.03 = 720.
JY is a 55 year-old breast cancer patient who is returning to clinic 7 days after completing her 4th course of CAF chemotherapy (cyclophosphamide/doxorubicin/fluorouracil). She complains of feeling tired and difficulty catching her breath when she gets up and moves around.

What is/are likely etiologies for her fatigue and shortness of breath?
A. She may have anemia occurring as a side effect of her chemotherapy
B. She may be experiencing cardiac toxicity occurring as a result of her
doxorubicin therapy
C. She may be having bladder toxicity as a result of her cyclophosphamide therapy
D. A and B
E. A and C
D. A and B

While cyclophosphamide can cause bladder toxicity in the form of
hemorrhagic cystitis, there is no evidence that HY has cystitis, and we are given no indication of bladder symptoms. Patients receiving chemotherapy can develop chemotherapy-associated anemia. Cardiac toxicity is a serious side effect than can be seen in patients receiving an anthracycline such as
doxorubicin.
From the question above: JY’s doctor orders an echocardiogram, which reveals an ejection fraction of 35 – 40%. Which of the following would not be appropriate with respect to JY’s chemotherapy?
A. She should receive 2 more cycles of CAF chemotherapy with a 50% dose reduction of doxorubicin
B. She should receive 2 more cycles of CAF chemotherapy with the addition of dexrazoxane
C. Her therapy should be changed to a regimen that doesn’t contain doxorubicin
B. She should receive 2 more cycles of CAF chemotherapy with the addition of dexrazoxane

JY appears to be having anthracycline-associated cardiac toxicity. Further
therapy with doxorubicin without the addition of a cardiprotectant puts her
at risk for more severe cardiac damage, therefore answer “A” is incorrect.
Either answer “B” or “C” are acceptable. Since anthracyclines are highly active against breast cancer, it may be reasonable to continue her CAF but
add the cardioprotectant agent dexrazoxane to prevent further heart
damage.
Which of the following toxicities is most often seen with epidermal growth factor
(EGF) receptor inhibitors?
A. Conjunctivitis
B. Bone marrow suppression
C. Peripheral neuropathy
D. Skin rash
D. Skin rash

In clinical trials of EGFR inhibitors, skin rash is a common side effect. These
drugs rarely cause neuropathies or severe myelosuppression.
CJ is 65 year old who has just been diagnosed with early stage colon cancer (stage II). As a treatment plan, what is the most appropriate recommendation?
a. Observation; as colon cancer is an indolent (relatively benign) disease, and patient is more likely to die with disease, than of it.
b. Surgery; based on the fact that the tumor is probably small, and data for chemotherapy use is scant
c. Folfox; based on the NEJM 2004 study that showed increased disease free survival when compared to 5fu-leucovorin
d. Irinotecan (Camptosar) + Cetuximab (Erbitux); based on the Bond 1 study that showed increased survival when compared to irinotecan alone
b. Surgery; based on the fact that the tumor is probably small, and data for chemotherapy use is scant

Surgery is mainstay of treatment for this stage of progression. If unchecked the Colon Cancer will grow rapidly leading to poor outcomes. Chemotherapy is not indicated at this stage of progression.
Choose the drug that is NOT appropriately linked with its dose limiting toxicity

A. Irinotecan (Camptosar) – Diarrhea
B. Oxaliplatin (Eloxatin) – Cardiac toxicity
C. Capecitabine (Xeloda) – Hand and Foot syndrome
D. Bevacizumab (Avastin) – Bleeding
B. Oxaliplatin (Eloxatin) – Cardiac toxicity

Dose limiting toxicity of Oxaliplatin is peripheral neuropathy
AJ is 65 year old with a PSA (prostate specific antigen) of 3ng/ml. What is the most appropriate course of action?
A. Observation
B. Further work-up with TRUS
C. Radical Prostatectomy
D. Estramustine (Emcyt)
A. Observation

TRUS is only recommended as a follow up for an abnormally elevated PSA.
Radical prostatectomy is not indicated as PSA is still within “normal” range.
Estramustine as a single agent does not have a role in the current care of prostate cancer patient.
Individuals with <10 year life expectancy should not be offered routine screening for prostate cancer.
A. True
B. False
A. True

Based on NCCN guidelines. Since disease progression is slow, patient is likely to die with prostate cancer, rather than, of prostate cancer.
B is 65 year-old male with stage IV prostate cancer. He is about to start therapy with LHRH agonist leuprolide (Lupron). He has metastatic disease. He is worried about the flare reaction associated with leuprolide. Based on a randomized, placebo controlled trial, the most appropriate recommendation is:
A. Leuprolide (Lupron) alone
B. Leuprolide (Lupron) + Flutamide (Eulexin)
C. Leuprolide (Lupron) + Finasteride (Proscar)
D. Leuprolide (Lupron) + Ketoconazole (Nizoral)
B. Leuprolide (Lupron) + Flutamide (Eulexin)

Leuprolide in combination with Flutamide has shown to increase overall survival and decrease the flare reaction associated with LHRH agonists.
Patient runs the risk of flare reaction with single agent leuprolide. Finasteride has been investigated as a primary prophylaxis agent and is not indicated for treatment of metastatic cancer Ketoconazole is indicated as a second line agent for hormone ablation, not indicated to reduce the flare reaction.
CA is a new patient in your clinic with prostate cancer who has disease progression
despite testosterone levels <20ng/ml. He is very much interested in continuing therapy to
increase his life expectancy. Considering his disease status, desire and your knowledge
of clinical studies, the most appropriate recommendation is:
A. Mitoxantrone (Novantrone) 12mg/m2 IV q3wks + Prednisone 5mg PO BID, since it has shown to increase overall survival
B. Mitoxantrone (Novantrone) 12mg/m2 IV q3wks + Prednisone 5mg PO BID, since it has shown to increase quality of life
C. Docetaxel (Taxotere) 75mg/m2 IV q3wks + Prednisone 5mg PO BID, since it has shown to increase overall survival
D. Docetaxel (Taxotere) 75mg/m2 IV q3wks + Prednisone 5mg PO BID, since it has shown to increase quality of life
C. Docetaxel (Taxotere) 75mg/m2 IV q3wks + Prednisone 5mg PO BID, since it has shown to increase overall survival.

Mitoxantrone + Prednisone has not been shown to increase survival. Docetaxel and Prednisone have been shown to increase overall survival. The patient, at this time, wants therapy that would increase overall survival and not quality of life.
Mrs. AK is a 46 year old female recently admitted to the inpatient bone marrow transplant (BMT) unit. She was admitted after she was seen in the BMT outpatient clinic complaining of pleuritic chest pain and fever and chills. She is currently Day + 78 after an allogeneic hematopoietic stem cell transplant from her sister. She has been receiving corticosteroids (prednisone) at a dose of 1 - 2 mg/kg /day for 3 weeks. A CT scan performed in the outpatient clinic revealed diffuse ground glass opacities and a halo sign.

What is the most likely cause of her pleuritic chest pain?
A. Invasive Aspergillosis
B. Candida albicans pneumonia
C. Pneumocystis carinii pneumonia
D. Cytomegalovirus
E. Steptococcus pneumonia
A. Invasive Aspergillosis

Candida albicans pneumonia is a rare entity and would not present
with pleuritic chest pain and a halo sign on CT. PCP is a possibility but would not be typically associated with a halo sign.
She was started on drug therapy for her infection, however continued to deteriorate. Her chest pain increased and she complained of some sinus symptoms. A repeat chest CT scan 4 days later demonstrated a fractured rib with bone invasion and her sinus CT indicated an aggressive invasive process. A biopsy is performed and is positive for Mucor. How would you modify her drug therapy regimen at this time, if at all?
A. No change – continue on the current treatment.
B. Add caspofungin (Cancidas) monotherapy – 70 mg IV loading dose, followed by 50 mg IV QD thereafter
C. Add voriconazole (Vfend) monotherapy – 6 mg/kg IV Q12H loading dose x 2 doses, followed by 4 mg/kg IV BID thereafter
D. Remove the catheter, Modify her drug therapy regimen to include ABLC (Amphotericin B lipid complex).
E. Administer combination antifungal therapy with voriconazole (Vfend) and ABLC (Amphotericin B lipid complex)
D. Remove the catheter, Modify her drug therapy regimen to include ABLC (Amphotericin B lipid complex).

Treatment of choice for Mucor out of these choices.
Mr. DFA is a 34-year-old male diagnosed with chronic phase chronic myeloid leukemia (CML). He is treated with imatinib (Gleevec) at a dose of 400 mg PO daily. Which of the following is the most likely complication of imatinib therapy?
A. Veno-occlusive disease
B. Edema
C. Peripheral neuropathy
D. Hyperleukocytosis
B. Edema

Most common complication from imatinib. VOD associated with
gemtuzumab not imatinib. Alopecia not the most common complication.
Hyperleukocytosis is not a side effect of imatinib.
Acute lymphocytic leukemia (ALL) is characterized by the following phases of
treatment:
A. CNS prophylaxis, maintenance therapy, remission induction therapy
B. Remission induction therapy and post-remission treatment
C. Remission induction therapy, consolidation/intensification, maintenance therapy, CNS prophylaxis
D. Remission induction therapy, CNS prophylaxis
C. Remission induction therapy, consolidation/intensification, maintenance therapy, CNS prophylaxis

ALL has 4 treatment phases for adults: remission induction, post-remission therapy (that can include transplant, etc) if no HSCT then maintenance therapy is administered, and CNS prophylaxis is administered.
PQ is a 23 year old female diagnosed with acute promyelocytic leukemia (APL). She started treatment with all-trans retinoic acid (ATRA) five days ago. Four days following the initiation of the ATRA her WBC count rises rapidly leading to intubation and admission to the intensive care unit. The most likely reason for her symptoms is:

A. Pneumonia
B. Retinoic Acid Syndrome (RAS)
C. Prolongation of the QTc interval
D. Pericardial effusion
E. Cardiomyopathy
B. Retinoic Acid Syndrome (RAS)
Which of the following statements most accurately describe the initial treatment of Candidemia in an immunocompromised host (with a history of acute leukemia) pending speciation and susceptibility testing?
A. Fluconazole (Diflucan) 200 mg PO QD until speciation then streamline
B. Caspofungin (Cancidas) 70 mg IV loading dose, then 50 mg IV QD until
speciation known then streamline
C. Anidulafungin (Eraxis) 50 mg IV QD until speciation then streamline
D. Posaconazole (Noxafil) 400 mg PO BID until speciation then streamline
E. Ketonconazole 200 mg PO QD until speciation then streamline
B. Caspofungin (Cancidas) 70 mg IV loading dose, then 50 mg IV QD until
speciation known then streamline

A is incorrect as we do not know species and susceptibility and in a
patient with leukemia they are likely to be exposed to fluconazole and therefore the species might be a non-albicans and the dose listed is incorrect. Dose of anidulafungin is incorrect for candidemia. Posaconazole would have similar issues to fluconazole and is not indicated in this setting. Ketoconazole would never be a choice in this patient population.
Treatment of choice for newly diagnosed acute lymphoblastic leukemia in the absence of a suitable donor for allogeneic hematopoietic stem cell transplantation is?
A. Cytarabine plus an anthracycline in the “7 + 3” regimen
B. Imatinib mesylate (Gleevec)
C. Alemtuzumab (Campath)
D. Weekly vincristine and anthracyclines in combination with prednisone
E. Gemtuzumab (Mylotarg)
D. Weekly vincristine and anthracyclines in combination with prednisone

Treatment of choice for ALL is vincristine plus and anthracycline
administered weekly x 4 doses. Steroids are continued for 28 days.
Cyclophosphamide may be added but by definition this is the best response to this question.
Mrs AH, a 65 year old female presents to the oncology clinic with a fever of 38.7oC (101.6o F). Her last chemotherapy regimen for lung cancer was administered 14 days ago. Her ANC is 310/mm3 and rising, she has a double lumen Port-A-Cath in place.
Her chest x-ray is clear and she has no obvious signs and symptoms of infection (no mucositis, no CVL tenderness, clear perirectal area). What treatment would be most appropriate based on her presenting signs and symptoms?
A. Cephalexin (Keflex) 500 mg PO QID + ciprofloxacin (Cipro) 500 mg PO BID
B. Ceftazidime (Fortaz) 2g IV Q8H monotherapy
C. Cefepime (Maxipime) 2g IV Q8H plus vancomycin 1g IV Q12H
D. Levofloxacin (Levaquin) 500 mg PO QD monotherapy
E. Ciprofloxacin (Cipro) 500 mg PO BID + amoxycillin/clavulanic acid
(Augmentin) 875 mg PO BID
E. Ciprofloxacin (Cipro) 500 mg PO BID + amoxycillin/clavulanic acid
(Augmentin) 875 mg PO BID

E (per the IDSA guidelines). Cephalexin is not an appropriate choice. Not sick enough to warrant IV Rx, no signs so vancomycin incorrect. Data to support levofloxacin in New England Journal of Medicine article, but not taught and also not in IDSA guidelines.
Mrs LW, a 43 year old female with febrile neutropenia status-post ALL induction chemotherapy is started on cefepime (Maxipime) 2 g IV Q8H. She becomes afebrile 48 hours after starting antibiotic treatment. Her blood cultures have all been negative however she is complaining of abdominal tenderness and a CT scan of her abdomen is performed. The findings are described as colon wall thickening and inflammation. What action would be most appropriate at this time?

A. Continue her cefepime (Maxipime) until she has been afebrile for 5 – 7 days, and then discontinue it
B. Discontinue the IV antibiotics and start oral metronidazole (Flagyl)
C. Stop the antibiotics as she is afebrile and has ANC recovery
D. Continue her current antibiotics and add an aminoglycoside and metronidazole (Flagyl)
D. Continue her current antibiotics and add an aminoglycoside and metronidazole (Flagyl)

The question does not give sufficient information as to whether she is not neutropenic. She has typhilitis and as such should be covered for gram negative pathogens – hence adding an aminoglycoside is appropriate as is adding anaerobic coverage.
BL is a 55 year-old male with newly diagnosed head and neck cancer. He is scheduled
to get neoadjuvant chemotherapy with cisplatin, methotrexate, and fluorouracil,
followed by a 2 week course of radiation therapy.
Which of the following statements is FALSE?
A. Neoadjuvant therapy is given prior to surgery
B. Neoadjuvant therapy is given after surgery
C. Neoadjuvant therapy may allow downstaging of the primary tumor
B. Neoadjuvant therapy is given after surgery

Neoadjuvant therapy is given prior to surgery to reduce the tumor size. This may reduce the extent of surgery needed, and may allow a tumor to be downstaged (i.e. a tumor that is stage III before therapy may decrease to a stage II after therapy, making it more amenable to surgicalresection.
LM is a 63 year-old female with newly diagnosed acute myeloid leukemia (AML) who was given induction therapy with cytarabine + idarubicin (7+3). She finished her chemotherapy yesterday and is generally feeling better. She has been getting up and walking around the nursing unit with assistance, but has fallen several times when attempting to walk on her own. Which of the following are concerns regarding LM?
A. She is at increased risk of infection, and should be allowed to ambulate only while wearing a mask.
B. She may be experiencing neurotoxicity, and should be closely monitored for worsening cerebellar symptoms.
C. She may be having bladder toxicity, and should be watched closely for the development of severe hematuria (blood in urine).
D. Both A and B
E. Both B and C
D. Both A and B

Because her counts will have fallen after receiving her leukemia
induction chemotherapy, she does have an increased risk of infection. Her falls are concerning for cerebellar toxicity, and she should be closely monitored for signs of worsening toxicity. C. is false, as the regimen LM received is not associated with bladder toxicity.
SK is a 65 year-old female diagnosed last year with locally advanced NSCLC (non small cell lung cancer). She received initial therapy with paclitaxel and carboplatin, which she
did not tolerate well due to severe arthralgias. Her therapy was changed to gemcitabine and cisplatin, which was better-tolerated. Disease staging evaluation done after completion of her last cycle of chemotherapy indicated that her lung cancer had not improved despite chemotherapy. She wishes to pursue further therapy and has been searching the internet, and asks about trying erlotinib (Tarceva).

Which of the following is TRUE regarding erlotinib (Tarceva)?
A. It targets the Bcr-Abl tyrosine kinase
B. It targets the epidermal growth factor receptor (EGFR) tyrosine kinase
C. It is indicated as first line therapy for NSCLC
D. Both A and B
E. Both B and C
B. It targets the epidermal growth factor receptor (EGFR) tyrosine kinase

It targets the EGFR tyrosine kinase, not the Bcr-Abl tyrosine kinase. It is indicated in patients who have failed at least one prior chemotherapy regimen. In several trials combining erlotinib with chemotherapy as first line therapy, there was no benefit seen with the addition of erlotonib.
Which of the following statements regarding tyrosine kinase inhibitors, such as imatinib (Gleevec) or erlotinib (Tarceva), is INCORRECT?
A. They are only available as oral formulations
B. Their side effect profile is very different from chemotherapy; rashes are often
seen.
C. They may penetrate tumors better than conventional chemotherapy
D. They are potentially curative therapy
E. None. All of the above are correct
D. They are potentially curative therapy

They are oral agents, and have a side effect profile that is generally much milder than that of many chemotherapy agents. Because they are small molecules, they do appear to be better able to penetrate into tumors compared to chemotherapy. To date, these agents have not demonstrated curative potential. The best outcomes have been with imatinib in patients with CML, and the responses are maintained only as long as patients remain on therapy; once therapy is stopped, it is likely that the disease will return.
JW is a 63 year-old White male with a past medical history significant for diabetes and GERD. He has a history of smoking 2 packs of cigarettes/day for 25 years. He presents with an increasingly painful cough that is occasionally flecked with blood (hemoptysis), and admits that he’s lost about 30 pounds without trying. On further questioning, he reports a positive family history of lung cancer (father).

Based on the information provided by JW, what risk factors does he have for developing lung cancer?

A. Smoking history
B. Exposure to ionizing radiation
C. Family history
D. Both A and B
E. Both A and C
E. Both A and C

His greatest risk factor is his smoking history (50 pack-year history).
While exposure to ionizing radiation does increase the risk of lung cancer, he
doesn’t report such an exposure. There does appear to be an increased risk in those with a family history.
Which of the following signs/symptoms that are present in JW make you suspicious for lung cancer?
A. Chest pain
B. Hemoptysis
C. Weight loss
D. Both A and B
E. Both B and C
E. Both B and C

JW reported coughing up bloody sputum (hemoptysis) and presented with weight loss. While chest pain may be a symptom of lung cancer, this was not included in his list of complaints.
AP is a 59 year-old White female who was recently found, on routine screening
mammography, to have a 2 cm spiculated mass in her left breast. On review of her reproductive history, she experienced menarche (onset of menstrual cycle) at age 12, has had two pregnancies, both resulting in live births (1st delivery at age 22), no history of hormone exposure, no family history of breast cancer. She is post-menopausal. She was referred for further evaluation of her breast mass. An ultrasound revealed a hypoechoic, irregular spiculated mass. A fine needle biopsy was positive for malignancy. She was referred to an oncologic surgeon, and underwent wide local excision; the pathology was positive for an infiltrating lobular carcinoma. She also underwent an axillary lymph node dissection, which revealed that 3 of 11 lymph nodes were positive, with limited extranodal extension. The ER (estrogen receptor) value was 135 fmol/mg, and PR (progesterone receptor) value was 96 fmol/mg.

AP’s surgeon referred her to a medical oncologist for adjuvant therapy. Which of the following is the most appropriate adjuvant chemotherapy for AP?
A. CMF (Cyclophosphamide, Methotrexate, Fluorouracil)
B. TAC (Docetaxel, Doxorubicin, Cyclophosphamide)
C. FEC (Fluorouracil, Epirubicin, Cyclophosphamide)
B. TAC (Docetaxel, Doxorubicin, Cyclophosphamide)

Adjuvant chemotherapy with taxane-containing regimens is
associated with improved outcomes.
What hormonal therapy should AP be given?
A. Fulvestrant (Faslodex)
B. Tamoxifen (Nolvadex)
C. Anastrazole (Arimidex)
C. Anastrazole (Arimidex)

As AP’s tumor is hormone-receptor positive, she should receive
adjuvant hormonal therapy. The ASCO technology assessment indicated that post-menopausal women with hormone-receptor positive tumors should receive an aromatase inhibitor. Therefore, anastrazole may be preferable to tamoxifen for adjuvant therapy. Fulvestrant is considered a second or third line agent for metastatic disease, and does not have a role in the adjuvant treatment of early stage breast cancer.
SG is a 63 year-old woman who was diagnosed in 1999 with an invasive ductal carcinoma after noticing a mass in her left breast. She underwent a modified radical mastectomy. Her total tumor size was 5.5 cm, and all 15 of her lymph nodes were positive. Her primary tumor was positive for ER/PR (estrogen/progesterone receptor), and HER-2 was overexpressed. Her metastatic workup was negative, and she received 4 cycles of AC (doxorubicin, cyclophosphamide) chemotherapy, followed by tamoxifen (Nolvadex). Following completion of her adjuvant chemotherapy, she received high dose chemotherapy followed by a stem cell transplant. She continued to be followed every 6 months, and had no evidence of recurrence until approximately 4 years after transplant, when she began to notice persistent left hip pain. A bone scan was done that revealed increased activity diffusely throughout the pelvis and lower spine, and she was diagnosed with recurrent breast cancer with metastatic bone involvement.

Which of the following would be most appropriate for SG’s metastatic disease?
A. Anastrazole (Arimidex)
B. Tamoxifen (Nolvadex)
C. Fulvestrant (Faslodex)
D. Raloxifene (Evista)
A. Anastrazole (Arimidex)

As SG’s tumor was hormone receptor positive, and her metastatic disease appears to be limited to bone, she would clearly benefit from hormonal
therapy. As she is post-menopausal, she is a candidate for an aromatase inhibitor. While tamoxifen could be used, comparative trials of either anastrazole or letrozole vs. tamoxifen in metastatic breast cancer have demonstrated a greater benefit in favor of the aromatase inhibitor. In this situation, tamoxifen should be reserved as a 2nd line agent. Fulvestrant is a 2nd/3rd line agent, and should be reserved should SG fail 1st or 2nd line therapy. Raloxifene is not indicated in the treatment of breast cancer; it is currently being evaluated to determine its efficacy as a preventive agent.
Mrs. AK is a 46 year old female currently admitted to the surgical intensive care unit. She was admitted following a motor vehicle accident 4 days ago and has significant
abdominal injuries. She is on broad spectrum antibacterial therapy; however despite the antibiotics she is spiking temperatures regularly to 39oC (102.2 oF). She is not
neutropenic, nor has she received prophylactic antifungal treatment. You are now called because her blood cultures are positive for Candida species.

Which if the following options is the best initial treatment for this patient based on her culture results and her clinical history?
A. Fluconazole (Diflucan) 400 mg PO/IV daily
B. 5-Flucytosine 50 mg/kg PO Q6H
C. Terbinafine (Lamisil) 250 mg PO QD
D. Amphotericin B (cAmB) 5 mg/kg/day plus voriconazole (Vfend) 200 mg PO/IV BID
E. Caspofungin (Cancidas) 70mg LD (loading dose) then 50 mg IV QD plus voriconazole (Vfend) 200 mg PO/IV BID
A. Fluconazole (Diflucan) 400 mg PO/IV daily
Mrs. AK is a 46 year old female currently admitted to the surgical intensive care unit. She was admitted following a motor vehicle accident 4 days ago and has significant
abdominal injuries. She is on broad spectrum antibacterial therapy; however despite the antibiotics she is spiking temperatures regularly to 39oC (102.2 oF). She is not neutropenic, nor has she received prophylactic antifungal treatment. You are now called because her blood cultures are positive for Candida species.

Which if the following options is the best initial treatment for this patient based on her culture results and her clinical history?
A. Fluconazole (Diflucan) 400 mg PO/IV daily
B. 5-Flucytosine 50 mg/kg PO Q6H
C. Terbinafine (Lamisil) 250 mg PO QD
D. Amphotericin B (cAmB) 5 mg/kg/day plus voriconazole (Vfend) 200 mg PO/IV BID
E. Caspofungin (Cancidas) 70mg LD (loading dose) then 50 mg IV QD plus voriconazole (Vfend) 200 mg PO/IV BID
A. Fluconazole (Diflucan) 400 mg PO/IV daily

The patient is not neutropenic nor has she been exposed to prior azole
therapy so we can assume that she will likely have a Candida albicans isolate and with the lack of exposure to prior azole therapy she will most likely have a
susceptible isolate. She is also clinically stable so monotherapy would be
appropriate. 5-Flucytosine should not be used as monotherapy for Candida
infections. Terbinafine is not indicated for systemic candidemia. Combination
therapy (ie option D and E) is not the standard of care. Were a combination to be used the one that has been evaluated in a randomized trial is fluconazole plus amphotericin B, not the 2 combinations listed. Furthermore the dose of
amphotericin B is incorrect as it should be 1 – 1.5 mg/kg/day.
Two days later the Candida species blood cultures are speciated. She is growing Candida kruseii from 4 separate blood culture bottles. Susceptibilities are pending. The patient remains febrile, although clinically stable. How would you modify her antifungal therapy at this time, if at all?

A. No change – continue on the current antifungal therapy chosen earlier.
B. Change to caspofungin (Cancidas) monotherapy – 70 mg IV LD (loading dose), followed by 50 mg IV QD thereafter
C. Change to voriconazole monotherapy (Vfend)– 6 mg/kg IV Q12H loading dose x 2 doses, followed by 4 mg/kg IV BID thereafter
D. Add amphotericin B (cAmB) to current therapy
B. Change to caspofungin (Cancidas) monotherapy – 70 mg IV LD (loading dose), followed by 50 mg IV QD thereafter

No change is incorrect. If fluconazole was chosen as it should in Q10 Candida kruseii it is inherently resistant to fluconazole and the treatment needs to be discontinued. The most well studied and effective agents in candidemia are the echinocandins and therefore the doses of caspofungin are correct. Voriconazole has been evaluated in candidemia but is not the treatment of choice at the current time. It does have activity against C.kruseii which is an advantage. Adding ampho B is not correct.
Mr. PA is a 65-year-old male diagnosed with chronic lymphocytic leukemia (CLL). He is treated with a fludarabine (Fludara)-based regimen, with the last dose 30 days ago. Complications of fludarabine that present abruptly, often several months after treatment has been completed, are well known. Which of the following is the most likely long-term complication of fludarabine therapy?

A. Secondary malignancy
B. Opportunistic infections such as Pneumocystis carinii pneumonia, cytomegalovirus, etc
C. Alopecia
D. Nausea and vomiting
E. Infertility
B. Opportunistic infections such as Pneumocystis carinii pneumonia, cytomegalovirus, etc

Purine analog therapy is known to result in opportunistic infections
due to its potent T-cell depleting activity. Secondary malignancies while they can occur with treatment are more common with alkylating therapy. Alopecia that far from treatment is not a recognized side effect. Nausea and vomiting that far from treatment is unlikely. Infertility – unlikely with fludarabine.
Acute myeloid leukemia (AML) is characterized by the following phases of
treatment:
A. CNS prophylaxis, maintenance therapy, remission induction therapy
B. Remission induction therapy and post-remission treatment
C. Remission induction therapy, consolidation/intensification, maintenance therapy
D. Remission induction therapy, CNS prophylaxis
B. Remission induction therapy and post-remission treatment

AML has 2 treatment phases: remission induction and post-remission
therapy (that can include transplant, etc). No CNS prophylaxis is administered. All other choices reflect treatment for acute lymphoblastic leukemia.
JW is a 49 year old female diagnosed with acute promyelocytic leukemia (APL). She has failed treatment with all-trans retinoic acid and is to start treatment with arsenic trioxide. Her WBC on the day she is to start arsenic trioxide is 1500/mm3.
Adverse effects and associated management associated with arsenic trioxide include:
A. Neutropenia: patient should be started on prophylactic antibacterial therapy
B. Retinoic Acid Syndrome (RAS): patient should be started on prophylactic corticosteroid therapy at the start of arsenic trioxide therapy
C. Prolongation of the QTc interval: a baseline EKG should be performed and a good medication history taken to evaluate for medications known to cause prolongation of the QTc interval.
D. Pericardial effusion: patient should be started on aggressive diuretic therapy prior to starting arsenic trioxide therapy
E. Cardiomyopathy: patient should have an echocardiogram or MUGA scan performed to ensure the ejection fraction is > 50% to allow arsenic trioxide therapy.
C. Prolongation of the QTc interval: a baseline EKG should be performed and a good medication history taken to evaluate for medications known to cause prolongation of the QTc interval.

Arsenic is reported to cause prolongation of the QTc interval.
Baseline estimation should be performed. Try to avoid medications know to cause prolongation of the QTc interval; Neutropenia may occur but prophylactic antibacterials are not warranted; RAS can/will occur, BUT prophylactic steroids is not the answer; pericardial effusion – more likely with ATRA or imatinib;
Cardiomyopathy – no with anthracyclines.
Mr BL, a 41 year old male is receiving treatment for T-cell ALL (acute lymphocytic leukemia). On day 25 of induction chemotherapy he is readmitted with febrile neutropenia and started on broad spectrum antibiotics. Persistent fever leads to a bronchoscopy and a transbronchial lung biopsy. He is febrile and has pleuritic chest pain. A CT scan of his chest reveals the development of bilateral airspace disease with nodules in each lung. Based on these findings what is the most likely cause of this patient’s infection?
A. Invasive Aspergillosis
B. Invasive Candidiasis
C. Crytptococcal infection
D. Fusarium species infection
E. Zygomycete infection
A. Invasive Aspergillosis

All the other pathogens do not typically present with nodules on a CT scan. The most likely cause in an immunocompromised patient is an infection by Aspergillus spp.
What is front-line treatment for newly diagnosed chronic phase chronic myeloid leukemia in the absence of a suitable donor for allogeneic hematopoietic stem cell transplantation?

A. Interferon (IFN) + cytarabine (ARA-C)
B. Busulfan (Busulfex)
C. Imatinib (Gleevec)
D. Gemtuzumab ozogamicin (Mylotarg)
E. Vincristine (Oncovin) + prednisone
C. Imatinib (Gleevec)

Treatment of choice for CML in CP is Imatinib.
Mrs AH, a 65 year old female presents to the oncology clinic with a fever of 38.7oC (101.6 oF). Her last chemotherapy regimen for lung cancer was administered 14 days ago. Her ANC (absolute neutrophil count) is 310/mm3 and rising, she has a double
lumen Port-A-Cath in place. Her chest x-ray is clear and she has no obvious signs and symptoms of infection (no mucositis, clear perirectal area). What treatment would be the most appropriate based on her presenting signs and symptoms?

A. Cephalexin (Keflex) (oral) + ciprofloxacin (Cipro) (oral)
B. Ceftazidime (Fortaz) (IV) monotherapy
C. Cefepime (Maxipime) (IV) plus vancomycin (IV)
D. Levofloxacin (Levaquin) (oral) monotherapy
E. Ciprofloxacin (Cipro) (oral) + amoxicillin/clavulanic acid (Augmentin) (oral)
E. Ciprofloxacin (Cipro) (oral) + amoxicillin/clavulanic acid (Augmentin) (oral)

E (per the IDSA guidelines). Cephalexin is not an appropriate choice. Not sick enough to warrant IV Rx, no signs so vancomycin incorrect. Data to support levofloxacin in New England Journal of Medicine article, but not taught and also not in IDSA guidelines.
Mrs LW, a 43 year old female with febrile neutropenia status-post ALL (acute lymphocytic leukemia) induction chemotherapy. She is started on cefepime (Maxipime) 2 g IV Q8H. She becomes afebrile 48 hours after starting antibiotic treatment. Her blood cultures have all been negative and she never developed obvious signs of infection. Her ANC (absolute neutrophil count) recovered to > 500/mm3 the day following admission and has remained > 500/mm3 for 3 consecutive days. What action would be most appropriate at this time?
A. Continue her cefepime (Maxipime) until she has been afebrile for 5 – 7 days, and then discontinue it
B. Discontinue the IV antibiotics and start oral ciprofloxacin (Cipro) and amoxicillin/clavulanic acid (Augmentin) therapy
C. Stop the antibiotics as she is afebrile and has ANC recovery
D. Continue her current antibiotics and add vancomycin to her therapy
C. Stop the antibiotics as she is afebrile and has ANC recovery

Per the IDSA guidelines. Patients who are afebrile by day 3 – 5 of antibacterial Rx and have an ANC > 500/mm3 for 2 consecutive days the recommendation is to stop antibiotics 48 hours after the patient is afebrile and ANC > 500/mm3.
YM is a 45 year-old male with a recent diagnosis of NHL (non hodgkins lymphoma). He is an entrepreneur with a fast-paced lifestyle. His business has just taken off. He is very upset about the diagnosis as he was told by his oncologist that he has to slow down and take care of his health first. He is scheduled to receive CHOP therapy (Cyclophosphamide 750 mg/m2 IV, Doxorubicin 50 mg/m2 IV, and Vincristine 1.4 mg/m2 IV with a max dose of 2 mg all given on Day 1, in addition to Prednisone (100 mg po daily on Days 1-5) all given q 21 days. YM is instructed to take his Prednisone as a part of his chemotherapy treatment in the morning before he comes into the infusion center for the rest of his chemotherapy.

Which of the following is the most appropriate antiemetic regimen to give before chemotherapy for the prevention of acute emesis in this patient?
A. Prochlorperazine (Compazine) 10 mg IV x 1 dose
B. Ondansetron (Zofran) 24 mg PO x 1 dose
C. Granisetron (Kytril) 1 mg IV and Dexamethasone (Decadron) 20 mg IV x 1 dose
D. Dronabinol (Marinol) 25 mg PO x 1 dose
E. Lorazepam (Ativan) 1 mg PO x 1 dose
B. Ondansetron (Zofran) 24 mg PO x 1 dose

This chemotherapy regimen has an emetic level of 4. It is the gold standard to use a combination of a 5HT3 antagonist + steroid for the prophylaxis of acute emesis in this setting. Since this patient is already on Prednisone as a part of the CHOP chemotherapy and the dose approximates Dexamethasone 20 mg, no additional steroid will be needed for antiemetic prophylaxis. In fact, it would be too much steroid to give to this patient if Answer C is chosen. Prochlorperazine and Dronabinol are not as potent as 5HT3 antagonists. Dronabinol needs to be given starting the day before chemotherapy, plus the dose is too high. Finally, Lorazepam is for anticipatory not acute emesis.
YM experienced severe nausea/vomiting 1 day after his first cycle of chemotherapy. What type(s) of emesis will he be likely to experience with subsequent cycles of
chemotherapy?
I. Acute
II. Delayed
III. Anticipatory
IV. Breakthrough

A. II only
B. III only
C. I and II
D. I, II and III
E. I, II, III and IV
E. I, II, III and IV

He experienced delayed emesis after the first cycle of chemo. Due to prior poor emetic control, this patient is at risk for anticipatory emesis in addition to acute, delayed, and breakthrough emesis which is common with this chemo regimen.
SS is a 46 year-old female with a diagnosis of ovarian cancer. She underwent surgical debulking and has been on Paclitaxel 135 mg/m2 IV followed by Carboplatin AUC 5 IV both on Day 1, repeated q 21 days. She tolerated the first cycle well and is in the clinic today to receive her second cycle. She has been premedicated with Dolasetron (Anzemet) 100 mg IV and Dexamethasone (Decadron) 20 mg IV, 30 min before the initiation of chemotherapy. She is a Gator fan and is excited talking about the basketball game she went to in Indianapolis. Fifteen minutes into her Carboplatin infusion, she starts to have some nausea and vomiting.

Which type of nausea/vomiting is she currently experiencing?
A. Anticipatory
B. Delayed
C. Breakthrough
D. Motion sickness
E. None of the above.
C. Breakthrough

She is experiencing breakthrough emesis despite being premedicated. She did fine with the 1st cycle of chemo, and the emesis occurs after (rather than before) chemo is given, so this is not anticipatory emesis. Since emesis occurs within 24 hours after chemo is given, it is not delayed emesis either (which occurs 24 hours after chemo). She is sitting down in the clinic receiving chemo, so her emesis is not the type related to motion sickness.
Which one of the following regimens is the most appropriate to treat her current
nausea?
A. Ondansetron (Zofran) 8 mg ODT PO
B. Aprepitant (Emend) 125 mg PO
C. Prochlorperazine (Compazine) 10 mg IV
D. Dexamethasone (Decadron) 20 mg PO
E. Palonosetron (Aloxi) 0.25 mg IV
C. Prochlorperazine (Compazine) 10 mg IV

Since she has received a 5HT3 antagonist and a steroid as premedication, (A) ondansetron, (E) palonosetron, and (D) dexamethasone that work through the same mechanisms will probably not provide a substantial clinical benefit, but potentially increase costs and overlapping toxicity. Also, there is limited data on the use of 5HT3 antagonists for breakthrough emesis. (B) Aprepitant is best used preventively before chemo is given. Prochlorperazine is an appropriate option since it works differently by blocking dopamine receptors and can be given prn for breakthrough emesis.
CC (chief complaint): “I have an increasing urgency to go the bathroom four to five times at night, and also have difficulty in urinating in the past 2 months.” HPI (history of present illness): MK is a 53 year-old white male who presented to the urology clinic (on 11/3/2000) for urinary symptoms and sexual dysfunction. About seven months prior to that, MK started to notice urinary urgency, nocturia, and difficulty in urination. He stated that these symptoms were getting more frequent and worse in the last two months. Otherwise, he denied any other discomforts. After questioning about the patient history and performing physical exam including digital rectal exam, the patient was asked to proceed to several other tests to rule out malignancies.
PMH (past medical history): GERD for 3 years
Hypertension for 5 years
Benign prostatic hyperplasia (BPH)
FHx (family history): father died of MI at age 69; (+) prostate cancer at age 60;
paternal grandfather (+)prostate cancer at age 63; mom died of CHF
ALLERGIES: None
Digital rectal exam: enlarged prostate; otherwise unremarkable.
PSA level: 2 ng/dl (normal range: 0-4 ng/dl)
Biopsies via TRUS revealed atypical hyperplasia in both peripheral and central zones in the prostate.
He underwent radical retropubic prostatectomy and was diagnosed with stage B prostate cancer with negative surgical margins.

Which of the following statement(s) is/are CORRECT?
I. Men with a normal PSA (prostate specific antigen) level can still develop prostate cancer.
II. The risk of developing prostate cancer in MK is strong because of his history of benign prostatic hyperplasia.
III. MK should undergo bilateral orchiectomy to improve overall survival.
Answer Choices:
A. Only I is correct
B. Only I and III are correct
C. Only III is correct
D. Only II and III are correct
E. I, II and III are correct
A. Only I is correct

The recent study showed that about 15 % of men aged between 50-75yo with normal PSA levels have been found to have biopsy-proven prostate cancer.
BPH has been shown to have no correlation with increased risk of prostate cancer. No published data to date showed that surgical castration can improve or prolong overall survival in early stage prostate cancer.
He received no further treatment and has been followed up with his urologist without further problems for several years. On 12/21/2004, MK was admitted to the ER for severe right hip pain and difficulty in urination. Bone scan also revealed increased bone uptake in right pelvis which was consistent with bone metastases of his prostate cancer. His PSA level was found to be 56 ng/dl. His medical history remained the same.
Which of the following statement(s) is/are INCORRECT with regard to the treatment options of metastatic prostate cancer in MK at this point?
I. The treatment intent of MK’s prostate cancer at this stage should be curative and aggressive.
II. MK can be offered bilateral orchiectomy.
III. MK can be offered hormonal therapy with abarelix (Plenaxis) indefinitely in addition to bicalutamide (Casodex) 50 mg po daily for 2-4 weeks to minimize tumor flares caused by abarelix.
IV. MK can be offered zoledronic acid (Zometa) once monthly.
A. Only I is incorrect
B. Only I and II are incorrect
C. Only I and III are incorrect
D. Only I and IV are incorrect
E. Only II and IV are incorrect
C. Only I and III are incorrect
Treatment goals for metastatic prostate cancer is palliative, ie, prolong survival and relieve symptoms, and improve pt’s QOL.First-line treatment for metastatic prostate cancer is hormonal ablation either done surgically (ie bilateral orchiectomy) or medically (LHRH agonist +/- antiandrogen such as bilcautamide). Abarelix is LHRH antagonist which competitively blocks LHRH receptors in the pituitary without causing receptor desensitization--> immediate inhibition of the hypothalamic-pituitary-gonadal axis. Therefore, patients who receive abarelix do not require antiandrogen therapy to decrease tumor flares in the beginning of treatment. Zoledronic acid, an intravenous bisphosphonate, is FDA-approved for prevention of progression of skeletal events in patients with bone metastasis including prostate cancer.
Two months after his diagnosis of metastatic prostate cancer and therapy, MK presented in the clinic to complain of frequent sweating during the day and night that requires to change his shirt three to four times daily and has hot flashes.
Which of the following medications is least likely to alleviate his symptoms?
A. Venalfaxine (Effexor)
B. Transdermal estrogen patches
C. Ketoconazole (Nizoral)
D. Clonidine
C. Ketoconazole (Nizoral)

Venalfaxine, clonidine and estrogen have been shown to relieve hot flashes caused
by hormonal depletion.
Fourteen months later, MK was re-admitted to the ER on 2/28/06 for weakness of bilateral lower extremities and significant back pain for 1 week. The patient also complained of progressive shortness of breath in the past two days. He was diagnosed with a pulmonary embolism.

Labs:
PSA (normal range 0-4 ng/dl)
12/21/04: 56 ng/dl
3/4/05: 1.8 ng/dl
5/17/05: 0.9 ng/dl
8/3/05: 2.7 ng/dl
11/23/05: 3.2 ng/dl
01/02/06: 5.8 ng/dl
2/28/06: 75 ng/dl

Testosterone (normal range 280-800 ng/dl)
540 ng/dl
less than 20 ng/dl
less than 20 ng/dl
---
---
---
less than 20 ng/dl

His current medications for his prostate cancer included goserelin acetate (Zoladex)
10.8 mg subcutaneous injection once every 3 months. His prostate cancer was thought to progress to hormone-refractory stage.
Which of the following statement(s) is/are CORRECT regarding the most appropriate treatment of MK’s prostate cancer at this point?
I. Chemotherapy therapy with mitoxantrone (Novantrone) + docetaxel (Taxotere) should be considered because it has been shown to improve overall survival.
II. MRI of spine should be ordered immediately to rule out spinal cord compression.
III. Goserelin acetate (Zoladex) should be withdrawn because one third of the patients in the studies have shown response to this approach.

A. Only I is correct
B. Only II is correct
C. Only I and II are correct
D. Only II and III are correct
E. I, II and III are correct
B. Only II is correct

MK’s clinical presentations strongly suspects spinal cord compression. The
immediate management for suspected spinal cord compression is to start the patient on steroids, and to order stat MRI spine. If anti-androgen withdrawal is considered, antiandrogen such as bicalutamide instead of LHRH agonist, should be withdrawn. Docetaxel + prednisone or docetaxel + estramustine, but not mitoxantrone +
docetaxel, have been shown to improve overall survival in pts with hormonal-
refractory prostate cancer.
DW is a 23-year-old female with a long history of irritable bowel syndrome since
teenager years who presented to an outside facility with complaints of crampy abdominal pain with intermittent constipation, diarrhea, as well as nausea and vomiting. The patient reports the symptoms were ongoing for the past six months, but progressively getting worse. She reports the pain occurred on a daily basis, described as crampy, 10/10 pain with no specific pattern. She did have a CT scan done at the outside hospital which revealed thickening of the sigmoid colon. This was followed up with a barium enema which showed obstruction at the sigmoid colon. She also underwent a colonoscopy which revealed a mass at 40 cm from the anal verge. This was biopsied and consistent with villous adenoma with high dysplasia. Repeat imaging was done at the hospital and confirmed the sigmoid mass and three suspicious areas were identified in the right lobe of the liver, which was further confirmed to be consistent with metastatic disease.
Pathology from the colon omentum, peritoneal nodule and primary tumor were all consistent with adenocarcinoma. She was diagnosed with stage IV colorectal cancer PMH (past medical history): 1) Irritable bowel syndrome since teenager; 2) Anemia. 3) Congenital blindness of the left eye from iris coloboma (a congenital (present since birth) defect of the iris of the eye.
PSH (past surgical history): 1) Partial colectomy, omentectomy, and excision of peritoneal nodule, as above. 2) C-section times one. 3) Appendectomy in 1994.
CURRENT MEDICATIONS: oxycodone 5-10 mg PO q4 hours prn for pain, St John’s wort for depression
FAMILY HISTORY: She has a grandmother with a history of breast cancer, a cousin with a history of ovarian cancer. No known family history of colon cancer.

Which of the following statement(s) regarding the diagnosis of colorectal cancer in DW is/are CORRECT?
I. Genetic counseling and the possibility of genetic testing should be provided in DW’s family.
II. The diagnosis of hereditary nonpolyposis colorectal cancer in DW is strongly suggested.
III. The diagnosis of familial adenomatous polyposis (FAP) in DW is strongly suggested.
Answer choices:
A. Only III is correct.
B. Only I and II are correct.
C. Only I and III are correct
D. Only II and III are correct
E. I, II and III are correct
C. Only I and III are correct

Based on DW’s clinical presentation, her colorectal cancer is located on the distal or left side (sigmoid), found to have adenoma with severe dysplasia at a young age, and a long history of irritable bowel syndrome at a young age; congenital eye disease (iris coloboma) all suggestive of high possibility of FAP syndrome. As high as 25% of patients with FAP do not have family history of colon cancer. For patients who have FAP, genetic counseling is strongly suggested.
Which of the following is the most appropriate regimen for treatment of DW’s newly diagnosed metastatic colorectal cancer?
A. DW should receive fluorouracil (5-FU) continuous infusion.
B. DW should receive irinotecan (Camptosar) plus fluorouracil (5-FU) plus leucovorin (IFL) plus bevacizumab (Avastin).
C. DW should receive oxaliplatin (Eloxatin) plus 5-FU plus leucovorin (FOLFOX).
D. DW should receive cetuximab (Erbitux) plus bevacizumab (Avastin).
B. DW should receive irinotecan (Camptosar) plus fluorouracil (5-FU) plus leucovorin (IFL) plus bevacizumab (Avastin).

Regimen of a, has never been shown to improve survival. Regimen b
has been shown to improve survival, especially when bevacizumab is added to the chemotherapy regimen, it has been shown to improve survival by 5 more months as compared to IFL; option c is good, but in her case, addition of bevacizumab to FOLFOX would be recommended to maximize her treatment outcomes; option d is considered as 3rd line (refractory disease) therapy for MCRC
A 51-year-old Caucasian female asks for your advice regarding treatment of her menopausal symptoms. She describes mild vasomotor symptoms and complains of symptoms consistent with marked vulvar and vaginal atrophy. These symptoms are new to her and she has not attempted treatment as of yet. Her medical history is unremarkable. What approach is recommended by the North American Menopause Society (NAMS)?
A. Start Prempro (conjugated estrogen/medroxyprogesterone) 0.3 mg and treat for the shortest duration possible
B. Start Premarin (conjugated estrogen) 0.3 mg and treat for the shortest duration possible
C. Initiate lifestyle modifications; such as daily exercise, relaxation techniques, and dietary changes; and Estrace (estradiol) cream 2 grams daily
D. Initiate lifestyle modifications, such as daily exercise, relaxation techniques, and dietary changes
C. Initiate lifestyle modifications; such as daily exercise, relaxation techniques, and dietary changes; and Estrace (estradiol) cream 2 grams daily

Systemic hormone replacement is not indicated since her vasomotor symptoms are mild. She is likely to achieve symptomatic relief with lifestyle modifications alone. If HRT were initiated, Prempro, a combination of estrogen and progestogen, would be recommended to protect against endometrial hyperplasia. Lifestyle modifications will not address her urogenital symptoms. These target vasomotor symptoms. Thus, lifestyle changes plus local estrogen therapy is most appropriate to target symptoms without exposing this woman to systemic estrogen therapy.
A 48-year-old Hispanic female requires pharmacotherapy for severe vasomotor
symptoms. She has a past medical history of breast cancer, HTN, obesity, and hyperlipidemia. Surgical history includes total abdominal hysterectomy.
VS: BP= 167/64, HR= 81, RR=18, Weight= 86.8 kg, Height= 169cm.
How should her symptoms be treated?
A. Start venlafaxine (Effexor), extended release 75 mg daily
B. Start gabapentin (Neurontin) 300 mg QHS and titrate to 900 mg daily
C. Start raloxifene (Evista) 60 mg daily
D. Start Prempro (conjugated estrogen/medroxyprogesterone) 0.3 mg and treat for the shortest duration possible
B. Start gabapentin (Neurontin) 300 mg QHS and titrate to 900 mg daily

Raloxifene acts as an estrogen antagonist with respect to vasomotor symptoms, which it can provoke. Prempro is contraindicated due to her history of breast cancer. Venlafaxine is contraindicated due to her uncontrolled hypertension. Gabapentin is the only agent that is not contraindicated, and its use is supported by recent clinical trial data.
You are counseling a women regarding appropriate use of alendronate (Fosamax). She asks how much calcium she needs to take with this medication. She tells you that she eats one serving of yogurt each day. Which of the following recommendations
would most closely meet the = recommendations for daily calcium intake?
A. Calcium carbonate 500 mg po TID
B. Calcium carbonate 600 mg po QD
C. Calcium citrate 500 mg po TID
D. Calcium phosphate 600 mg po QD
A. Calcium carbonate 500 mg po TID

Her calcium intake from diet is 650 mg. Her target calcium intake from diet plus
supplements is 1200 mg elemental calcium. Converting the above salts to elemental calcium content reveals that answer A, 1500 mg calcium salt x 0.40 = 600, would increase calcium intake to 1250 mg per day. Other answers would not provide sufficient elemental calcium. It is also critical not to significantly exceed 1200 mg since the recent WHI calcium study indicated a 17% increase in the risk of renal
calculi with calcium supplementation.
A 52 year-old Asian women with a history of HTN, pulmonary embolism, smoking, breast cancer, wrist fracture, and depression has a T-score of -1.7 at the femoral neck.
VS: BP= 137/64, HR= 78, RR=18, Weight= 56.8 kg, Height= 163cm.
Allergies: penicillin (rash), alendronate (rash, shortness of breath)

Which of the following treatment strategies is most appropriate?
A. Recheck BMD in one year to determine when pharmacotherapy is appropriate
B. Initiate Actonel (risedronate) 35 mg PO once weekly
C. Initiate Evista (raloxifene) 60 mg PO once daily
D. Initiate Forteo (teripartide) 20 mcg SC once daily
D. Initiate Forteo (teripartide) 20 mcg SC once daily

Pharmacological therapy is indicated given history of fracture, suppressed T-score, and risk factors. She had a significant reaction to alendronate, thus preventing use of all bisphosphonates. Her history of VTE mitigates against using a SERM, although it would likely decrease her risk of breast cancer in the future. Thus, despite its high acquisition cost, teriparatide is the best answer.
Regarding oral contraceptives, which of the following is FALSE?
A. Minipills are less effective at preventing pregnancy than combination pills
B. More patients stop taking oral contraceptives due to anxiety about side effects as compared to those who discontinue due to actual side effects
C. Low dose combination pills are designed to provide reduced doses of estrogen and progestin as compared to typical combination pills
D. Birth control pills should be manipulated/changed based on the progestin component and not the estrogen component
E. The pearl index is used as an indicator of the effectiveness of birth control at preventing pregnancy
C. Low dose combination pills are designed to provide reduced doses of estrogen and progestin as compared to typical combination pills

Low dose combo pills provide a reduced dose of estrogens but not progestins. All other statements are true
Which of the following is NOT an absolute or relative contraindication to the use of oral contraceptives?
A. Thromboembolic disorder
B. Cerebrovascular accident
C. Coronary artery disease
D. Diabetes mellitus
E. All of the above are absolute or relative contraindications
E. All of the above are absolute or relative contraindications

All of these are absolute or relative contraindications (see pg 18 of handout)
MT is a 20 year-old female who recently began taking Ortho-Novum 1/35
(norethindrone 1 mg, ethinyl estradiol 35 mcg). After 6 weeks she complains of some mild spotting during the first week of each pack she has taken. Which of the following would be the most appropriate recommendation at this time?
A. Change to Norinyl 1/80 (norethindrone 1 mg, mestranol 80 mcg)
B. Change to Ortho-Novum 1/50 (norethindrone 1 mg, mestranol 50 mcg)
C. Change to Yasmin (ethinyl estradiol 30 mcg, drospirenone 3 mg)
D. Change to minipill such as Ovrette (norgestrel 0.075 mg)
E. Continue Ortho-Novum 1/35 for now
E. Continue Ortho-Novum 1/35 for now

Spotting usually resolves within 3 months, so need to allow more time on current pill. If doesn’t resolve after 3 months then consider change.